You are on page 1of 40

A 3-month-old boy is brought to the clinic by his mother because "ever since the child was

born he has had constant tearing from both eyes." She tells you that when the baby
wakes up in the morning there is a small amount of watery discharge at the medial corner
of his eyelids and that she must wipe the child's eyes multiple times throughout the day.
The child is developmentally normal and is reaching his normal milestones. On physical
examination the baby is able to fix and follow you with either eye. Extraocular movements
are full. The pupils are round and reactive and there is no relative afferent papillary
defect. The conjunctiva, sclera, and cornea are normal. There is a bit of dry crust on the
medial side of the left lower lid. The most appropriate next step in management is to
A. admit the patient for intravenous antibiotics
B. instruct the mother to apply warm compresses to both eyes 4 times a day
C. instruct the mother to perform nasolacrimal duct massage 2-3 times a day
D. refer the patient to an ophthalmologist for nasolacrimal duct probing
E. take cultures and prescribe topical antibiotics
Explanation:
The correct answer is C. Nasolacrimal duct obstruction is a very common condition in
pediatrics. The nasolacrimal ducts are usually patent at 3-4 weeks. Tearing and crusting
are the typical features of an obstruction. The primary treatment is nasolacrimal duct
massage. Approximately 90% of nasolacrimal duct obstructions spontaneously resolve
in the first year of life.
IV antibiotics (choice A) are appropriate for preseptal or postseptal orbital cellulites but
the patient does not present with red, swollen, and tender lids.
Warm compresses (choice B) are the appropriate treatment for a chalazion. This patient
did not present with a nodular, non-infectious swelling in the eyelids.
Since most obstructions spontaneously resolve, surgical intervention and probing are
usually not recommended until age 6 months (choice D).
When a patient has a nasolacrimal duct obstruction and there are signs of a localized
infection (mucoid discharge with matting of the eyelashes) then topical antibiotics have
a role. However, if the patient is just experiencing tearing, antibiotics are not indicated
(choice E).
A 31-year-old woman comes to the office because of a 1-week history of early nasal
congestion, which has recently progressed to headaches, facial pain and right ear
pressure. She has had no fever, but her congestion has been increasing. She has no
other past medical history and has taken only over-the-counter decongestants in the past
week. These have not helped to relieve her symptoms. Her temperature is 37.0 C (98.6
F). There is evidence of purulent green nasal discharge on nasal speculum examination.
Otoscopic examination reveals normal tympanic membranes with a normal cone of light
bilaterally. Her lungs are clear. The diagnostic sign most likely to be found in this patient is
A. a heart murmur
B. impaired transillumination of the sinuses
C. photophobia
D. swelling of maxillary sinuses
Explanation:
The correct answer is B. This patient has acute sinusitis. The most common etiology is
infection and the organisms most responsible are rhinovirus, H. influenza, S.
pneumoniae, and influenza virus. The hallmark of acute sinusitis compared with sinus

inflammation is total ostial obstruction. Once obstructed, fluid accumulates and


becomes infected. The diagnosis is suggested by clinical signs and symptoms. The 5
findings that have a combined likelihood ratio of greater than 7 for acute sinusitis are:
mucopurulence, toothache, impaired transillumination, poor response to decongestants,
and tenderness.
The presence of a heart murmur (choice A) is not relevant to sinusitis, but may be an
associated finding in many young people or can be present in acute rheumatic disease,
which this patient does not have.
The presence of photophobia (choice C) along with neck stiffness and headache are
highly suggestive of meningitis, not sinusitis.
Although tenderness and congestion of sinuses are common, swelling maxillary
sinuses(choice D), even with massive fluid accumulation is not seen, since the bony
containment of the sinuses prevents any significant swelling.
A 51-year-old woman who is admitted to the hospital for a third cycle of chemotherapy for
non-Hodgkin's lymphoma (NHL) reports gradual development of blurry vision. Her past
medical history is significant for NHL, which is widely metastatic to the neck, chest, and
abdomen. Vital signs are normal. Physical examination reveals mild bilateral papilledema.
Extraocular movements are intact bilaterally. The pupils are equal, round, and normally
reactive to light. Vision is 20/200 bilaterally. Review of an eye examination performed 6
months ago reveals that vision was 20/40 bilaterally. The sinuses and ears are normal on
examination. A neurologic examination is normal. Laboratory studies show:

An MRI of the orbits demonstrates edematous extraocular muscles without fatty


replacement. An MRI of the brain is normal. There is mild optic nerve edema bilaterally.
The most important immediate next step is
A. intravenous antibiotics
B. intravenous heparin
C. intravenous steroids
D. orbital decompression surgery
E. standard NHL chemotherapy
Explanation:
The correct answer is C. This patient has clinical and MRI evidence of non-Hodgkin's
lymphoma (NHL) metastatic to the extraocular muscles. This is causing secondary
compression of the optic nerve which manifests as blurry vision. Immediate intravenous
steroid administration is critical to reduce inflammation and relieve optic nerve pressure.
Antibiotics (choice A) are not necessary as there are no clinical signs of infection.
Sinusitis and/or orbital cellulitis are common causes of unilateral optic nerve
compression. This patient has clinical and MRI evidence of non-Hodgkin's lymphoma
(NHL) metastatic to the extraocular muscles. This is causing secondary compression of
the optic nerve. Immediate intravenous steroid administration is critical to reduce
inflammation and relieve optic nerve pressure.
Anticoagulation with heparin (choice B) is not necessary since her symptoms are not

related to a stroke.
Orbital decompression surgery (choice D) has no role in the treatment of this patient.
Metastatic non-Hodgkin's lymphoma (NHL) is the primary cause of extraorbital muscle
enlargement and secondary optic nerve decompression. Chemotherapy (choice E)
should continue on an urgent basis. Intravenous steroid therapy is the most important
immediate treatment to reduce inflammation and relieve optic nerve pressure.

A 2-year-old girl is brought to the emergency department by her mother because of


respiratory distress. The mother reports that the child went to bed with mild upper
respiratory symptoms and then woke up in the middle of the night with sudden respiratory
distress. On entering the examination room, you notice that the child is in respiratory
distress and is sitting on the examination table, drooling saliva from her mouth. The nurse
is hooking the child up to pulse oximetry and electrocardiogram leads. The next
appropriate step in the management of this child should be to
A. examine the child in flat position
B. immediately alert the operating room and anesthesiologist
C. immediately intubate the child
D. obtain a chest x-ray
E. obtain her temperature
Explanation:
The correct answer is B. This child has a classical appearance of acute epiglottitis,
which is a life-threatening illness. Epiglottitis in children is usually associated with
sudden difficulty in breathing, sore throat, and dysphagia. Acute epiglottitis in children is
usually because of Haemophilus influenza. The child classically appears in severe acute
respiratory distress of sudden onset, leaning forward in a sitting position with drooling of
saliva from the mouth. The voice is muffled and the strider is softer than with croup. This
emergency should be diagnosed immediately and help must be obtained by calling an
anesthesiologist and a senior pediatrician. The operating room must be alerted for an
emergency cricothyroidotomy, if emergency intubation fails. An oxygen mask should be
held close to the child's face and one should wait by the child's side until senior help
arrives. Prepare to intubate the child with the smallest endotracheal tube. This is an
essential life-saving step and no delay should be allowed in getting this child intubated
by a senior physician.
This child should not be examined in a flat position as it might worsen the respiratory
distress. Acute epiglottitis is characterized by the swelling of the epiglottis. Laying the
child in a flat position to examine the oral cavity and the pharynx might obstruct the
airway because of the swollen epiglottis (choice A).
Intubating this child (choice C) is going to be challenging and difficult, and should be
done by only an experienced anesthesiologist. The child should be immediately taken to
the operating room for appropriate intubation, and if that fails, then a cricothyroidectomy
is necessary. Repeated failed attempts to intubate may result in airway edema and may
be life threatening.
A chest x-ray (choice D) is not essential in the diagnosis of this child. This can be
performed only when a pulmonary infection is suspected and in this child, it might delay
the diagnosis and therapy.
Obtaining the temperature (choice E) is not essential when acute epiglottitis is
suspected. Once acute epiglottitis is ruled out, one should examine for the evidence of a
peritonsillar abscess causing high fever. Before proceeding with the differential

diagnosis, acute epiglottitis must be excluded.

A 14-year-old boy is brought to the office because of a 5-minute history of a nosebleed.


The mother tells you that she was driving him to school and she looked over at him and
he had blood streaming down his face. They were passing by your office so they decided
to stop by. This is the first nosebleed that he has ever had. He denies any history of
trauma. He has been a patient of yours since he was 3 years old, and he has always
been very healthy. His blood pressure is 110/80 mm Hg and pulse is 65/min. He has mild
epistaxis. Physical examination is otherwise unremarkable. After giving him a tissue and
telling him to pinch his nostrils, you should give him an ice pack and
A. advise him to tilt his head back
B. obtain hematologic laboratory studies
C. recommend that he keep his nasal passages moist
D. tell him to continuously blow his nose to evacuate the blood
E. test his blood for cocaine
Explanation:
The correct answer is C. Since this is his first nosebleed (epistaxis), it seems as if this
boy has a common nosebleed, which is often due to forceful nose picking and dry nasal
passages. The blood typically comes from the anteroinferior portion of the nose and it is
usually controlled within 5-10 minutes by applying steady pressure to the mobile part of
the nose. If this is ineffective, a topical vasoconstrictor or cautery with silver nitrate or
electrocautery may be indicated.
It is inappropriate to advise him to tilt his head back (choice A) because the blood will
drip down the nasopharynx and into the esophagus and gastrointestinal tract. There is
also a risk for aspiration.
It is unnecessary to obtain hematologic laboratory studies (choice B) at this time.
Nosebleeds are extremely common and since this is his first one, it is most likely due to
a dry environment or nasal picking, as opposed to a bleeding disorder. Liver disease is
a consideration in a person with severe epistaxis, however, he has no other signs or
symptoms associated with liver disease.
He should not be told to continuously blow his nose to evacuate the blood (choice D).
The pressure should stop the bleeding. The increased pressure generated when he
blows his nose will only lead to further bleeding.
While cocaine use is associated with nosebleeds, it is inappropriate to test his blood for
cocaine (choice E) at this time. He has no other signs or symptoms of cocaine use and
this is only his first nosebleed (or so he says). If you somehow feel that he may be using
drugs, it is appropriate to ask his mother to leave the room and then ask him about his
drug use. It is incorrect to test his blood without asking him about his drug use.

A 42-year-old man with a history of alcoholism comes to the clinic because of a "swollen
tongue." He feels as if his tongue is taking up more room in his mouth and it is sometimes
mildly painful. He drinks a half a bottle of vodka per night and smokes a pack of
cigarettes per day. He denies intravenous drug use and has never had a sexually
transmitted disease. He laughs when you ask him about his sexual activity, stating that he
"hasn't had sex in ages." Physical examination shows a slightly enlarged, smooth pale
tongue with a loss of filiform papillae. The remainder of his examination is unremarkable.

Laboratory studies show:

The most appropriate next step is to


A. advise him to decrease his alcohol consumption and perform no further studies
B. determine vitamin B12 level
C. perform a brush biopsy of the tongue for cytologic evaluation
D. prescribe fluconazole, orally
E. refer him to a surgeon for an incisional biopsy
Explanation:
The correct answer is B. This patient has glossitis, which is most likely due to anemia.
He is an alcoholic with a macrocytic anemia (decreased hemoglobin, hematocrit and an
elevated MCV), and therefore, vitamin B12 levels should be determined. It is very likely
that he has a vitamin B12 deficiency.
While it is always appropriate to advise him to decrease his alcohol consumption, you
need to determine his vitamin B12 levels, so performing no further studies (choice A) is
incorrect. If he is vitamin B12 deficient, vitamin replacement is indicated.
A brush biopsy of the tongue for cytologic evaluation (choice C) and an incisional biopsy
(choice E) are usually performed to evaluate a lesion for malignancy. This patient has a
generalized glossitis without a specific lesion.
Fluconazole (choice D) is prescribed for candidiasis, which typically presents with a
white coating on the tongue. This patient's physical findings are inconsistent with this
diagnosis.

A 31-year-old woman comes to the office with a 5-day history of a red, painful right eye.
She complains of photophobia, tearing, and decreased visual acuity and denies any
history of trauma. She has had several similar episodes in the past. On examination, her
visual acuity is 20/30 in the right eye, both pupils react normally, the conjunctiva is
diffusely injected, and there is a watery discharge from the right eye. Fluorescein staining
shows uptake in the right cornea, but the cornea is otherwise clear. The left eye is normal.
The most likely cause of these findings is
A. acute angle closure glaucoma
B. allergic keratoconjunctivitis
C. bacterial conjunctivitis
D. herpes simplex virus keratitis
E. subconjunctival hemorrhage
Explanation:
The correct answer is D. Herpes simplex keratitis should be suspected in patients with
recurrent symptoms of unilateral pain, redness, and photophobia in the presence of an
epithelial staining defect. Under slit lamp biomicroscopy these staining defects often
take on a "dendritic" appearance, but without a slit lamp they may appear as just
fluoroscein uptake. These patients should be referred to an ophthalmologist for
treatment. Never prescribe topical corticosteroids for patients with suspected herpes

simplex keratitis.
Angle closure glaucoma (choice A) could be consistent with recurrent episodes, but not
with this clinical presentation. Patients with angle closure glaucoma will present with
poorly reactive pupils, marked decrease in visual acuity, and corneal edema making the
cornea unclear.
Allergic diseases (choice B) are most commonly bilateral and are associated with a
mucoid discharge and are less likely to cause pain, photophobia, and epithelial defects.
Bacterial conjunctivitis (choice C) rarely produces an epithelial defect, is often
associated with copious mucoid discharge, and would not be expected to be recurrent.
Subconjunctival hemorrhage (choice E) is a benign clinical finding. In this case, it would
not explain pain, photophobia, tearing, or decreased visual acuity.

A 4-year-old boy is admitted to the hospital with right eyelid swelling and redness.
Approximately 3 days prior to admission he developed nasal discharge, fever, and then
24 hours prior, developed the right eyelid swelling and erythema. He has a history of mild
asthma and his vaccinations are up to date. His temperature is 38.9 C (102 F), blood
pressure 114/68 mm Hg, and pulse is 78/min. There is mild proptosis on the right.
Extraocular motor examination is remarkable for difficulty moving the right medially, pupils
are equally round and reactive to light. No lymphadenopathy is present. Laboratory
studies show: leukocyte count 18,000mm3, hematocrit 35%, blood urea nitrogen 10
mg/dL, creatinine 0.6 mg/dL, sodium 135 mg/dL, and potassium 4.1 mg/dL. Intravenous
cefuroxime therapy is initiated. The most appropriate next step in management is to
A. administer amphotericin B, intravenously
B. discharge him after 24 hours of antibiotics
C. order a CT scan of the orbits with intravenous contrast
D. prepare him for surgical exploration of the right orbit
E. prepare him for surgical exploration of the sinuses
Explanation:
The correct answer is C. This patient has a cellulitis of the right eyelid. One of the most
important things to find out in order to guide therapy is whether the infection is preseptal
or postseptal. The orbital septum can be thought of as running along the anterior aspect
of the bones of the orbit separating the superficial tissues from the orbital contents
themselves. Some signs of postseptal infection that this patient has are
ophthalmoplegia and proptosis. The risk of visual loss and spread to deeper structures
such as the cavernous sinus are why this diagnosis is important to make, as opposed to
a superficial orbital cellulitis, which is fairly easily treated. It is also important to look for
any abscesses that might need to be drained. CT scanning can help to elucidate many
of these.
Amphotericin B (choice A) is an important tool in the treatment of fungal infections such
as mucormycosis. Infection with this fungus is extremely deadly and should be
considered in the differential diagnosis of any patient with a severe sinus infection.
There is often evidence of devitalized tissue and patients often have some sort of
relative immune suppressive disease such as diabetes or chronic renal insufficiency.
There is nothing in this case to suggest that amphotericin should be added at this point.
Discharge of the patient should be only after adequate treatment, which may take more
than 24 hours (choice B). Intravenous antibiotics can be discontinued and switched to
oral after the patient has been stable and afebrile for 24 hours.
Surgical exploration of the sinuses or orbit (choice D and E) should not be performed

prior to a CT scan. If an abscess or other signs of more severe infection are seen on the
scan, then surgical debridement may be warranted.

A 48-year-old woman is brought to the emergency department by her husband because


of an "excruciatingly painful left eye." They were at the movies when she began to
complain of blurry vision, a severe headache, and she had to run to the restroom to
throw-up. The pain "settled into her eye" on the ride over to the hospital. She is now
complaining of seeing "halos" around lights. Her ophthalmologic history is significant for
several dendritic herpetic ulcers over the past few years. Physical examination shows
conjunctival hyperemia with an edematous left eyelid. The cornea appears "steamy" and
the pupil is fixed and mid-dilated. The left eye is tender and firm on palpation and
tonometric testing reveals an intraocular pressure (IOP) of 67mm Hg. Immediate
management would be to
A. administer mannitol, intravenously
B. order a CT scan of the head
C. perform a lumbar puncture
D. perform a peripheral iridotomy
E. treat the patient with corticosteroids, topically
Explanation:
The correct answer is A. This patient most likely has acute angle-closure glaucoma,
which is an ocular emergency that requires immediate treatment to prevent blindness. It
is characterized by a rapid increase of IOP, often occurring when the pupil is dilated
(such as in a dark movie theater). The symptoms include severe eye pain, blurry vision,
a headache, nausea, and vomiting. The physical examination usually shows a tender,
firm eye with conjunctival hyperemia, lid edema, a hazy cornea, and a fixed, mid-dilated
pupil. Immediate treatment includes mannitol (to reduce vitreous volume),
acetazolamide and topical beta-blockers, such as timolol (to block aqueous production),
and pilocarpine (to facilitate aqueous outflow). Ophthalmologic consultation should also
be sought as soon as possible. A peripheral laser iridectomy is the definitive procedure
and is usually performed after the IOP is controlled.
A CT scan of the head (choice B) is indicated in the evaluation of papilledema, not acute
angle-closure glaucoma. Papilledema, which is optic disc swelling caused by elevated
intracranial pressure, typically presents with a headache, nausea, vomiting, and
transient visual obscurations.
A lumbar puncture (choice C) is performed after a CT scan of the head as part of the
evaluation of papilledema, not acute angle-closure glaucoma. If the CT scan is normal
and the opening pressure is elevated the most likely diagnosis is a pseudotumor
cerebri. This typically affects young, obese women and is treated with weight reduction
and acetazolamide.
A peripheral iridotomy (choice D) is the definitive procedure for acute angle-closure
glaucoma, but it is usually performed after the IOP is controlled with mannitol,
acetazolamide, timolol, and pilocarpine.
Corticosteroids (choice E) are not typically included in the initial management of acute
angle-closure glaucoma and they are contraindicated in patients with dendritic herpetic
ulcers.

A 64-year-old farmer with poorly controlled diabetes mellitus is brought to the


7

emergency department, because his wife noticed facial asymmetry. His wife
reports that he does not take insulin injections regularly and his blood sugar is
often very high. The patient has recently noticed some discharge from his right
ear. During the conversation, it is apparent that there is some amount of
hearing loss. His temperature is 37.0 C (98.6 F), blood pressure is 120/90 mm
Hg, and pulse is 84/min. His blood sugar is 254 mg/dL. Head and neck
examination reveals 2 skin lesions on the face. Neurological examination of
the cranial nerves reveals slight paresis on the right side of the face. Local
examination of the right ear shows purulent discharge, and some amount of
granulation tissue. Otoscopic examination of the right ear, after irrigation with
fluid, shows granulation tissue in the external auditory canal. Examination of
the left auditory canal was normal. The next step in the management of the
condition in his ear is to
A. administer antibiotic eardrops
B. begin intravenous penicillin and an aminoglycoside
C. counsel him about regular care of both of his ears
D. prescribe oral penicillin
E. prepare him for surgical debridement of the ear
Explanation:
The correct answer is B. The condition described in this farmer is malignant
external otitis. Malignant external otitis is an unusually virulent form of
external otitis caused by Pseudomonas aeruginosa, which occurs particularly
in poorly controlled diabetics. It produces pain, purulent otorrhea, hearing
loss, and characteristically, the external auditory canal contains granulation
tissue. There is destruction of the bone of the external auditory canal, and
the osteomyelitis may spread along the base of the skull to the midline or to
the opposite side. Facial paralysis often occurs due to involvement of the
tubes. Therapy is directed towards the causative organisms. While waiting
for the organisms to be isolated, the patient should be admitted to the
hospital for therapy with intravenous systemic penicillin and aminoglycoside.
Once the appropriate culture results are obtained, the organisms should be
identified and the sensitivity should be followed to change the antibiotics.
Apart from treating this condition with intravenous antibiotics, the diabetes
mellitus should be controlled well with insulin or an oral antidiabetic regimen.
Antibiotic eardrops are not sufficient to treat this aggressive form of otitis
externa (choice A). This patient needs to be admitted to the hospital for
intravenous antibiotic therapy. Failing to do this, will result in the progress of
the disease process.
Although care of the auditory canals is very essential in diabetics, mere
regular cleaning (choice C) is not sufficient enough to prevent or treat
malignant otitis externa. Hence, the patient should be advised before leaving
the hospital about taking care of his both ears, but by itself is not sufficient to
treat malignant otitis externa.
Oral penicillin is not adequate antibiotic therapy for this form of aggressive
otitis externa (choice D). Hence, this patient should be admitted to the
hospital for intravenous antibiotic therapy.
Surgical intervention (choice E) is not of much help in malignant external
otitis. Therapy is directed towards the causative organisms and usually
requires 6 weeks of oral ciprofloxacin or intravenous aminoglycoside and
synthetic penicillin.

A 31-year-old man returns to the clinic for the third time in 4 months complaining of right
ear pain. Previously, the patient was diagnosed with otitis media and treated successfully
with antibiotics. The patient confirms that he has taken the entire prescribed course of
antibiotics. His past medical history is significant for occasional lower back pain for which
he occasionally takes ibuprofen. Vital signs are: temperature 37 C (98.6 F), blood
pressure 110/70 mm Hg, pulse 64/min, and respirations 12/min. Physical examination
shows a white, amorphous debris in the right middle ear. There is conductive hearing
loss. The remainder of the examination is normal. The next step in managing this patient
is to
A. order a bone scan with SPECT images of skull
B. order a CT scan of the temporal bones
C. prescribe a 1-month course of erythromycin
D. prescribe a 1-month course of imipenem
E. prescribe prednisone
F. send him to an otolaryngologist for myringotomy
Explanation:
The correct answer is B. Recurrent otitis media in an adult leads to the suspicion for
cholesteatoma, which is a congenital or acquired epidermoid inclusion cyst of the middle
ear. The physical exam confirms this diagnosis given white, amorphous debris in the
right middle ear, which is desquamated epithelial debris. It often destroys the external
auditory canal bone and leads to a perforation of the tympanic membrane. It typically
requires surgical therapy.
Preoperatively, most surgeons obtain a detailed temporal bone CT to evaluate the
cholesteatoma. A bone scan with SPECT images of the skull (choice A) is useful to look
for osteomyelitis, but osteomyelitis would be low on the differential diagnosis in this
patient given the lack of fever.
Antibiotics such as erythromycin (choice C) may treat the current infection, but will not
treat the underlying cholesteatoma leading to the infection.
Antibiotics such as imipenem (choice D) may treat the current infection, but will not treat
the underlying cholesteatoma leading to the infection.
Prednisone (choice E) would not be helpful in this patient. Workup for cholesteatoma
should commence with a CT of the temporal bones.
Myringotomy (choice F) is indicated for recurrent ear infections, but the physical exam
findings in this patient suggests cholesteatoma.

A 26-year-old man comes to the office because of a 2-day history of a hoarse voice. He
developed a sore throat, runny nose, and a cough about a week ago that have basically
"gone away." He has not been to any concerts or sporting events recently where he says
that he usually "loses" his voice. He denies any difficulty breathing. His temperature is 37
C (98.6 F), blood pressure is 120/80 mm Hg, pulse is 60/min, and respirations are
14/min. Physical examination shows diffuse erythema of the larynx and slight
engorgement of the vocal cords. The remainder of the examination is unremarkable. The
most appropriate next step is to
A. obtain a laryngeal biopsy
B. obtain a laryngeal culture

C. order a radiograph of the neck


D. prescribe oral antibiotic therapy
E. recommend resting the voice and inhaling humidified air
Explanation:
The correct answer is E. This patient has laryngitis, which is an inflammation of the
laryngeal mucous membranes that is most likely caused by a viral infection. The history
and physical examination are usually enough to establish a diagnosis. If an exudate
were present, diphtheria, mononucleosis, candidiasis, and a streptococcal infection
should be considered. Treatment includes resting the voice and inhaling humidified air.
A laryngeal biopsy (choice A) is not indicated at this time in this case of laryngitis. A
biopsy is indicated if a lesion is visualized or suspected.
A laryngeal culture (choice B) is not indicated at this time in this case of laryngitis, which
is most likely viral in origin. It should be obtained if you suspect a bacterial infection or
an etiology other than a viral infection.
A radiograph of the neck (choice C) is indicated when there is severe airway narrowing
with marked hoarseness, a fever, and drooling. This patient does not have difficulty
breathing, a fever, or severe airway narrowing.
Antibiotics (choice D) are indicated if a bacterial infection is present. This patient has a
history consistent with an acute viral infection and therefore, antibiotics are not indicated
at this time.

A 5-year-old boy is admitted to the hospital because of his increasing irritability and fever.
The mother reports that the child has been having upper respiratory symptoms for the
past week. In the last few days, the child has been constantly rubbing his left ear and has
been increasingly irritable. The patient has had multiple ear infections in the past, that
were treated with oral antibiotics. Following recurrent episodes of these ear infections, the
child was advised to take a prophylactic, single dose of amoxicillin at bedtime. The child
has been taking these antibiotics regularly for the past 3 months. On examination, the
child is found to have a fever of 38.8 C (101.8 F). Examination of the right ear does not
reveal any abnormalities. The examination of the left ear is uncomfortable, but the
external auditory meatus appears normal. The tympanic membrane is examined after the
removal of the cerumen and is noted to be hyperemic, bulging with indistinct anatomical
landmarks. Light reflex is diminished and there is limited mobility on pneumatic
insufflations. Some amount of middle ear effusion is also noticed. The most appropriate
next step in the management is to
A. advise a tympanostomy tube placement
B. continue antibiotic prophylaxis with amoxicillin
C. prescribe a 10-day course of oral antibiotics
D. recommend decongestants, along with oral antibiotics
E. start intravenous antibiotic therapy
Explanation:
The correct answer is A. Otalgia, fever, irritability, previous or coexisting upper
respiratory tract infections, ear rubbing, and feeding problems are common presenting
symptoms of otitis media. Many of these episodes are viral in origin. The most common
bacterial pathogens are Pneumococcus, Haemophilus influenza, and Moraxella
catarrhalis. Diagnosis involves adequate observation of the tympanic membrane, which
might reveal hyperemia, bulging, indistinct landmarks, diminished light reflex, and

10

limited mobility on pneumatic insufflations. Treatment of acute otitis media with the
above symptoms and signs is by means of oral antibiotics. For recurrent acute otitis
media, antibiotic prophylaxis (such as a single dose of amoxicillin at bedtime) should be
considered. Referrals for discussion of a tympanostomy tube placement should be
considered if there is chronic bilateral effusion for more than 3 months in duration,
unilateral effusion for more than 3 months in duration, language development delay,
hearing loss of more than 20 decibels, or failure of antibiotic prophylaxis. This particular
child has a history of recurrent acute otitis media treated with antibiotic prophylaxis.
Failure of antibiotic prophylaxis requires tympanostomy tube placement.
Continuation of antibiotic prophylaxis (choice B) is not going to treat the recurrence of
acute otitis media in this child and hence, is not advisable.
Oral antibiotics (choice C) are indicated in the treatment of acute otitis media.
Amoxicillin is the first antibiotic of choice and should be given for 10 days. But for
recurrent episodes of acute otitis media, a tympanostomy tube placement is indicated
rather than mere oral antibiotics.
Decongestants (choice D) are not indicated in the resolution of acute otitis media,
although they might be needed for any associated conditions.
Intravenous antibiotics play no role (choice E) in the treatment of recurrent acute otitis
media. The surgical option should be advised to the parents.

A 57-year-old man comes to the emergency department because of a severe headache.


The headache came on suddenly as he was leaving the light show at the planetarium. He
also has right eye pain and nausea and he vomited twice during the car ride over to the
hospital. He has no significant past medical history. He has had other headaches in the
past, but has never experienced anything like this. His temperature is 36.7 C (98.0 F),
blood pressure is 130/90 mm Hg, pulse is 75/min, and respirations are 18/min. Physical
examination shows a tender red right eye with a partially dilated pupil, but is otherwise
unremarkable. The most appropriate next step is to
A. administer glucocorticoids, intravenously
B. administer sumatriptan, intramuscularly
C. give him oxygen therapy, 100% for 15 minutes
D. measure intraocular pressure
E. order a CT scan of the head
F. order an erythrocyte sedimentation rate
G. perform a lumbar puncture
Explanation:
The correct answer is D. This patient with a headache and a red eye either has acute
glaucoma or a cluster headache. The partially dilated pupil is more consistent with acute
glaucoma. Acute glaucoma, which is caused by increased intraocular pressure, often
presents with abdominal pain, nausea, vomiting, and a headache. It is an
ophthalmologic emergency that may lead to blindness, so this diagnosis should be
included in your differential when a patient presents with these other symptoms. The
patient usually has eye pain, but it may be overshadowed by the other symptoms. The
diagnosis is established by measuring intraocular pressure. You should call for an
immediate ophthalmologic consultation and possibly administer acetazolamide, topical
beta blockers, mannitol, and pilocarpine. A cluster headache is an episodic headache
that typically presents with a few short headaches a day for a few weeks that is

11

associated with periorbital pain, reddening of the eye, and lacrimation. A partially dilated
pupil is not a common finding.
Administer glucocorticoids, intravenously (choice A) is the appropriate treatment to
prevent blindness in a suspected case of temporal arteritis. Temporal arteritis typically
presents with a unilateral headache, myalgias, jaw pain, fever, and weight loss. A
tender, reddened temporal artery is often found. Visual changes may occur and
blindness is a feared complication. This patient does not have any of the usual systemic
symptoms associated with temporal arteritis. Also, the red eye with a dilated pupil is
inconsistent with this diagnosis. The diagnosis of temporal arteritis is established with a
temporal artery biopsy. The erythrocyte sedimentation rate (choice F) is typically
elevated in temporal arteritis
Sumatriptan (choice B) is the treatment for a migraine headache, which typically
presents with a throbbing headache, nausea, vomiting, photophobia, and functional
impairment. An aura consisting of hallucinations and scotomas sometimes occurs
before the headache. A partially dilated pupil is inconsistent with a migraine.
Oxygen therapy, 100% for 15 minutes (choice C), is the treatment for a cluster
headache. Sumatriptan may also be given to shorten the attack.
A CT scan of the head (choice E) is important if an intracranial hemorrhage is
suspected. An intracranial hemorrhage may present with a headache and nuchal rigidity,
seizures and confusion. A red, tender eye is not a typical finding.
A lumbar puncture (choice G) should be performed in a suspected case of meningitis
and possibly in a suspected intracranial hemorrhage, if the CT scan fails to show the
bleed. Meningitis often presents with a headache, nuchal rigidity, and photophobia.
Fever and a rash may be present. A tender, red eye with a partially dilated pupil is not
typically found in meningitis or an intracranial hemorrhage.
A 26-year-old man comes to the office because of a 4-month history of nasal discharge,
facial pain, and fevers. He was in the office a few months ago, when his symptoms
began, and you recommended over-the-counter decongestants and antihistamines. His
symptoms initially improved but over the past couple of months have again worsened. He
comes to the office now because his fevers and nasal discharge are persistent and the
discharge has become purulent. His medical history is otherwise unremarkable with only
a knee arthroscopy last year secondary to a medial meniscus tear. Physical examination
shows mucopurulent discharge in the region of the middle meatus and diffusely
congested sinus mucosa. Transillumination of the sinuses is impaired. The most
appropriate next step is to
A. determine white blood cell count
B. obtain a facial MRI
C. order a plain radiograph of face
D. order a sinus CT scan
E. perform a inferior meatus puncture
Explanation:
The correct answer is D. This patient has chronic sinusitis. The definition of chronic is a
sinus infection greater than 3 months. The most common etiology is infectious and the
organisms most responsible are rhinovirus, H. influenza, S. pneumoniae, and influenza.
The hallmark of acute sinusitis compared with sinus inflammation is total ostial
obstruction. Once obstructed, fluid accumulates and becomes infected. The diagnosis is
suggested by clinical signs and symptoms although certain imaging studies are very
useful. The only imaging study to consider is a sinus CT. If positive, the maxillary or

12

ethmoid sinuses will be opacified.


The localized nature of the infection often times means that there will usually be no
elevation in the systemic white blood cell count (choice A). If this test were normal, it in
no way rules out the presence of a sinus infection.
A facial MRI (choice B) actually offers much less sensitivity that a CT scan since the
high resolution imaging of the sinus material is confusing to the diagnosis. CT is
superior because opacification is a highly abnormal finding.
A plain radiograph of the face (choice C) offers no utility in the diagnosis of chronic
sinusitis because the visualization of all of the sinuses is inadequate.
A maxillary sinus irrigation-inferior meatus puncture is performed for diagnostic and
therapeutic purposes (choice E). Since this is an invasive procedure, a rigid nasal
endoscopy can be used to directly visualize the nasal discharge in situ and should be
performed before the puncture. For this reason, to avoid the invasive puncture study, a
sinus CT should be performed.

A mother brings her 8-month-old son to the clinic because of a fever. The mother reports
that for the last few days he has had a runny nose and has been more irritable than
usual. He was treated for otitis media in the right ear approximately 3 weeks ago with a 7day course of amoxicillin. He has had no other episodes of otitis media and has no other
significant past medical history. He was born full term by normal spontaneous vaginal
delivery with no perinatal complications. His mother reports that his immunizations are
up-to-date for his age, but she does not know if he has received the heptavalent
pneumococcal vaccine. He has 2 older brothers and attends day care with 12 other
children. Both of his parents smoke inside the house. His temperature is 39.1 C (102.4
F). Physical examination shows that his right tympanic membrane is erythematous and
bulging. He has a small amount of mucoid secretions in each nasal cavity. The remainder
of his physical examination is normal. In addition to treating the acute infection, you
should advise the mother that the most important step in preventing future ear infections
is to
A. avoid smoking inside the house or around the child
B. ensure that the child receives the 23-valent pneumococcal vaccine
C. have the child take a 4-week course of amoxicillin
D. refer the child to an otolaryngologist for myringotomy and tube placement
E. remove the child from day care
Explanation:
The correct answer is E. A large body of evidence exists to show that children in group
day care are significantly more likely to develop otitis media after an upper respiratory
tract infection compared to those in home care. One study showed that the rate of
tympanostomies and adenoidectomies is 59-67% higher in children younger than 3 who
attend day care. In addition, having older siblings, as this child does, has also been
shown to significantly increase the risk of otitis media.
The data on the effect of passive smoke on the prevalence of otitis media in children are
inconclusive. One study showed a higher incidence of tympanostomy tubes, chronic and
recurrent otitis media, and otorrhea in children whose mothers smoke (choice A), but
many other studies have failed to find a significant relationship between passive smoke
and otitis media.
The 23-valent pneumococcal vaccine (choice B) is not effective in children younger than
2. However, a recent study showed that in children who received one or more doses of

13

the heptavalent pneumococcal vaccine, the incidence of total invasive pneumococcal


disease was reduced by 89.1% and the need for myringotomy and tubes was reduced
by 20.1%.
In patients with otitis media who fail to respond to amoxicillin or develop a recurrent
infection after amoxicillin, amoxicillin-clavulanate, cefuroxime, cefprozil, or ceftriaxone
are the antibiotics of choice. Evidence exists that a child with persistent otitis media with
effusion (3 months or more) is much more likely to respond to a 1-month course of
amoxicillin-clavulanate than amoxicillin (choice C).
This child does not meet the criteria for myringotomy and tympanostomy tube
placement (choice D). For recurrent otitis media, the criteria for tube placement are 3 or
more episodes of otitis media in 6 months or 4 or more episodes in 12 months.

A 65-year-old woman comes to the office because of "blurry vision." She states that her
vision has been getting worse for years and that it is now difficult for her to read the
newspaper. Further questioning reveals that she has stopped driving at night because of
difficulty with her vision. Her past medical history is remarkable for well-controlled
hypertension, peptic ulcer disease, obesity, and anxiety. When examining the patient's
visual acuity in the office, it is clinically most helpful to
A. check each eye separately with appropriate spectacle correction
B. exam the patient's color vision and stereo visual acuity
C. record the visual acuity in both eyes simultaneously to best evaluate the
functional vision of the patient
D. record the visual acuity without spectacle correction to most accurately assess
the patient's functional vision
E. use a near card instead of Snellen (distance) visual acuity
Explanation:
The correct answer is A. It is important to evaluate the visual acuity of each eye
separately with the appropriate refractive correction. Refractive errors are generally not
considered a medical reason for decreased vision. Eliminating or reducing this factor will
best allow the clinician to assess the patient's vision. Remember that the testing of
visual acuity is a subjective test and that many factors need to be considered such as
lighting, patient effort, and patient concentration.
Color vision and stereovision are more specific methods of evaluating visual acuity.
These tests are used in more special circumstances in ophthalmology clinics. They
would not be more helpful than Snellen visual acuity in this instance (choice B).
Record the visual acuity in both eyes simultaneously to best evaluate the functional
vision of the patient (choice C) is incorrect because recording the visual acuity of both
eyes is rarely helpful. Patients may have 20/20 vision with one eye and no light
perception in the other eye and still have 20/20 vision if both eyes are tested together.
Often patients do not notice marked decreased vision if only one eye is affected.
Again, visual acuity without correction can be misleading (choice D).
Near cards are helpful in the hospital setting, but they are not as accurate as a
standardized Snellen vision measured at 20 feet (choice E). Also, remember that
accurate near visual acuity needs appropriate near correction (bifocal). Thus, checking a
near visual acuity in a presbyopic patient without their glasses can be misleading (for
example a hospitalized patient who has forgotten to bring their glasses with them to the
hospital).

14

A 4-year-old girl is in the hospital for a tonsillectomy. During the preoperative evaluation,
she points to a bump on her neck. The girl's mother reports that the bump has been there
as long as she can remember. The patient has no significant past medical history and
takes no medications. Vital signs are normal. Physical examination reveals a soft, mobile,
midline neck mass. The patient denies any pain during palpation of the mass.
Ultrasonography of the neck reveals a 1.0 x 1.0 x 2.2 cm midline neck cyst with a thick
wall just below the hyoid bone. The thyroid is normal in appearance and size. The next
step in managing this patient is
A. ampicillin plus gentamicin
B. laryngoscopy
C. percutaneous drainage
D. surgical removal
E. a thyroid biopsy
F. a thyroid stimulating hormone (TSH) assay
Explanation:
The correct answer is D. This patient is presenting with a thyroglossal duct cyst. These
cysts are remnants of the primitive thyroid, which descend from the foramen cecum at
the base of the tongue, to below the level of the hyoid bone where the adult thyroid
resides. Thyroglossal duct cysts account for 70% of all congenital neck anomalies. The
typical appearance is a mobile mass on the anterior neck. Although they are benign
lesions, they may become infected and painful. They must be surgically extirpated to
avoid recurrence. Care must be taken during preoperative evaluation with an ultrasound
to exclude an ectopic thyroid gland, which may occur along the thyroglossal duct.
Antibiotics (choice A) are necessary if the cyst is infected. The patient, however, denies
any symptoms such as pain that would indicate an infected cyst.
Laryngoscopy (choice B) will provide direct visualization of the airway. Since the cyst is
external to the larynx, it will not be visualized and thus a laryngoscopy is not indicated.
Percutaneous drainage (choice C) is insufficient treatment for a thyroglossal duct cyst,
since the cyst will very likely recur without surgical extirpation.
Thyroid biopsy (choice E) is unnecessary as the thyroid is normal on ultrasound.
A thyroid stimulating hormone (TSH) assay (choice F) is unnecessary as there are no
clinical signs of thyroid dysfunction. Thyroglossal cysts are not associated with thyroid
dysfunction.

A 33-year-old woman who you began treating for depression 2 weeks earlier with
amitriptyline comes to the emergency department because of a "migraine headache",
"fuzzy vision", nausea, and one episode of vomiting. She was seen 4 days ago in the
emergency department for similar complaints and states that "the doctor couldn't find
anything wrong with me." Review of her records shows a normal physical examination,
normal CBC, and normal CT of the head. She was given oxycodone/acetaminophen for
her headaches and sent home. Now, the visual acuity is 20/20 in the right eye and 20/200
in the left eye. Examination of the right eye is normal, however examination of the left eye
reveals a non-reactive pupil to light or accommodation. The left pupil is fixed at 5 mm and
there is redness of the conjunctiva. The optic nerves appear normal in both eyes. The
most appropriate next step in the management of this patient is to
A. begin a trial of sumatriptan therapy

15

B. counsel her about depression


C. obtain a STAT ophthalmology consult
D. order an MRI of the brain and orbits
E. send VDRL and FTA-ABS studies
Explanation:
The correct answer is C. This patient has angle closure or acute glaucoma. These
patients often have unilateral headaches from referred pain in the distribution of the
ophthalmic division of the trigeminal nerve. They experience blurry vision from corneal
edema. Perhaps the most important clinical sign is a mid-dilated fixed pupil in the
context of a red eye and headache. Most patients also complain of ocular pain. In acute
glaucoma, the optic nerve often appears normal, unlike chronic open angle glaucoma
where cupping and pallor of the nerve are present. This patient was also recently placed
on a tricyclic antidepressant that has anticholinergic effects, promoting dilation of the
pupil and placing the patient at increased risk for angle closure glaucoma. Urgent
ophthalmic consultation is indicated for the treatment of this patient.
A migraine headache is not the right diagnosis. Although this patient does have a
unilateral headache, visual disturbance, nausea, and vomiting, a migraine headache
would not be responsible for a fixed mid-dilated pupil. Treatment with antimigraine
therapy (choice A) would not be indicated.
Depression would not cause the clinical signs presented above. Attributing this
presentation to depression (choice B) would miss the diagnosis and potentially cost this
patient her vision.
An MRI of the brain and orbits (choice D) would not be helpful in angle closure
glaucoma.
An investigation for syphilis (choice E) is not likely to be positive. An Argyll-Robertson
pupil is one that accommodates to near but does not react to light. This is a chronic
finding of tertiary neurosyphilis. The optic nerve would demonstrate pallor.

A 71-year-old man with mild hypertension and high cholesterol comes to the office
complaining of 2 weeks of intermittent vertigo with each episode lasting about 2-4 hours.
He also reports hearing a low frequency buzzing, which is constant but waxes and wanes
in intensity. He tells you that over this time he has been having trouble hearing while in
noisy areas such as in restaurants or temple gatherings. Physical examination is normal.
Vertigo is not exacerbated by changes in head position. The most appropriate
management of this patient is to
A. begin diazepam therapy
B. begin hydrochlorothiazide therapy
C. begin meclizine therapy
D. begin scopolamine therapy
E. recommend physical therapy
Explanation:
The correct answer is B. This patient has Meniere's syndrome (endolymphatic hydrops).
Meniere's syndrome is characterized but intermittent vertigo lasting about 1-8 hours with
associated hearing loss, aural pressure, and tinnitus. Symptoms tend to wax and wane.
It differs from benign positional vertigo in that the symptoms do not necessarily worsen
with positional changes. In addition, positional vertigo doesn't have the same associated
factors as Meniere's syndrome does. The treatment is a low salt diet and HCTZ. If the

16

patient is resistant to medical therapy, surgery to decompress the endolymphatic sac


can be a last resort.
Diazepam (choice A) is a useful treatment to ablate an acute episode of acute vertigo
but is not a first line therapy for Meniere's syndrome.
Meclizine (choice C) is an antihistamine, which is useful in the management of less
severe attacks of vertigo.
Scopolamine (choice D) is sometimes used in a transdermal preparation to be beneficial
in the management of chronic vertigo. The anticholinergic side effects can limit its
usefulness.
Physical therapy (choice E) is becoming more important in the management of vertigo.
It is thought to help the enhance CNS ability to compensate for labyrinthine dysfunction.
Recently, use of specific head maneuvers has been incorporated into the management
of vertigo.

A 9-year-old girl is brought to the clinic because she has felt "sick" and has been unable
to go to school for the past 2 days. She complains of a headache, congestion, rhinorrhea,
and double vision. Her past medical history is remarkable for recurrent otitis media 2
years ago for which she eventually received bilateral myringotomy tubes. She lives at
home with her mother and grandmother who are both cigarette smokers. Her temperature
is 38.8 C (101.8 F), pulse is 120/min, respirations are 20/min, and visual acuity is 20/20 in
both eyes. Physical examination shows tympanic membranes with evidence of previous
surgery, but are otherwise normal, an erythematous oropharnyx with exudation, and slight
exophthalmos of the left eye. On the left, the ocular examination also demonstrates
periorbital edema, injection of the conjunctiva, trace restriction of extraocular movements,
and an afferent pupillary defect. The right eye is normal. The rest of her physical exam is
unremarkable. At this time the most correct statement about her condition is:
A. A CT of the head should be ordered
B. Gentamycin eye drops treat most cases, but patients need close observation
C. Hospitalization and intravenous antibiotics are indicated
D. Oral antibiotics are the first line of treatment
E. Otitis media is a common cause of this condition
Explanation:
The correct answer is C. This patient has orbital cellulitis. Patients present with lethargy,
fevers, signs and symptoms consistent with sinusitis, proptosis of the globe, restriction
of extraocular movements, periorbital edema, injected conjunctiva, and afferent pupillary
defects. Hospitalization and intravenous therapy should be started at once as this
condition is potentially fatal, especially if the cavernous sinus becomes involved. In
children, most cases are a result of ethmoid sinusitis and periosteal abscess formation.
ENT and ophthalmology consults are necessary to follow this patient in the hospital.
Surgical drainage of abscesses are considered in patients who are not responding to
antibiotic therapy or if visual acuity/pupil function is deteriorating.
A CT of the head (choice A) would not be as helpful as a CT of the orbits. Orbital scans
should be ordered with axial and coronal views.
Gentamycin eye drops (choice B) are of no value. Orbital signs such as restriction of
ocular movement, proptosis, and an afferent pupillary defect should not be overlooked
to treat a "red eye."
Oral antibiotics (choice D) are not adequate treatment for orbital cellulitis. Preseptal
cellulitis may be treated with oral antibiotics. Patients with preseptal cellulitis have

17

periorbital erythema and edema, but do not display the orbital symptoms described
above. The globe is usually white and quiet with preseptal cellulitis.
Otitis media (choice E) is not a cause of orbital cellulitis. As mentioned above, sinusitis
is the most frequent source of infection. Severe otitis media can erode through to the
sigmoid sinus and cause venous sinus thrombosis resulting in cranial nerve palsies and
increased intracranial pressure.

A 42-year-old woman comes to the office because of the sudden onset of "blurry vision"
in her left eye. She says that she wears glasses regularly to see distant objects, but now
she is seeing "blurry" in both near and distant objects. She has moderate pain, but denies
any nausea or vomiting. She does not have any significant past medical history and does
not take any medications. The most appropriate method for performing the
ophthalmologic examination is to
A. ask the patient to look directly at your nose as you approach with the
ophthalmoscope
B. hold the ophthalmoscope in your left hand when examining the patient's left
eye
C. hold the ophthalmoscope 6 inches in front of your face as you approach the
patient's eye
D. hold the ophthalmoscope up to your right eye as you examine the patient's left
eye
E. press the ophthalmoscope against the patient's cheek and keep the hand that
is holding the instrument in the air
Explanation:
The correct answer is B. When performing an ophthalmologic examination you should
hold the ophthalmoscope in your left hand, up to your left eye when examining the
patient's left eye, and in the right hand when examining the patient's right eye. If you
picture this, you will realize that this allows you to stand on the patient's side and invade
their personal space as little as possible. If you hold the instrument in your right hand,
up to your right eye and examine the patient's left eye, you need to stand directly in front
of them, and basically on top of them. This is totally inappropriate. It will make you and
the patient very uncomfortable.
You should tell the patient to look straight ahead and stare at a distant target, not to look
directly at your nose as you approach with the ophthalmoscope (choice A).
You should not hold the ophthalmoscope 6 inches in front of your face as you approach
the patient's eye (choice C). You should function as a single unit with the instrument by
holding it against your own forehead as you approach her. This allows you to have the
most coordinated actions, and allows you to visualize the red reflex as you approach.
You invade the patient's personal space by holding the ophthalmoscope up to your right
eye as you examine the patient's left eye (choice D). You should hold the instrument in
your left hand to your left eye when examining the patient's left eye, and then switch to
your right hand and eye to examine the patient's right eye. Also, you cannot really be as
stable and visualize the eye as well if you hold the instrument in your left hand, up to
your right eye as you examine the patient's left eye. And even then you invade their
personal space more than if you held it in left hand to your left eye to examine the
patient's left eye.
You should not press the ophthalmoscope against the patient's cheek and keep the
hand that is holding the instrument in the air (choice E). The instrument should be held

18

against your forehead and you can gently hold the hand with the scope against the
patient's cheek, but not the instrument, for stabilization. You are not very stable if you
keep the hand with the instrument in the air.

A 10-year-old girl is admitted to the pediatric floor with a sore throat and high fever. She
was feeling well until 2 days ago when the mother noticed that she was coughing, febrile,
and complaining of a sore throat. She was seen in the office by your partner who started
her on oral penicillin. But on the second day, the mother noticed that the girl's fever went
up to 38.7 C (101.6 F) and she brought her to the emergency department. The mother
also reported that other children at school were having similar complaints recently. In the
emergency department a throat swab is taken and, because of the high fever, she is
admitted to the pediatric floor. She is started on intravenous synthetic penicillin as
antibiotic therapy. On the second day of hospitalization, you are called to the pediatric
floor because the girl is now complaining of severe throat pain with radiation to the ears.
On approaching the patient, you notice that she is drooling saliva and appears to be in
distress. The nurse reports that her fever is 39.1 C (102.4 F), blood pressure is 110/70
mmHg, and pulse is 124/min. You notice that she has a muffled voice. Examination of the
throat reveals a unilateral swelling of the palate and anterior pillars with displacement of
the tonsils downward and medially. The uvula is moved away from the involved side. The
right side is the most involved and there are occasional white patches on the left tonsil.
There is a swollen anterior cervical node on the right side. The nurse comes back to
report that the throat culture taken on the day of admission is positive for Group A beta
hemolytic Streptococcus, and sensitive to penicillin. At this point the mother also reports
that the girl has had 3 episodes of sore throat in the past year, and that they all
responded to oral antibiotics. You advise the mother and the patient that an emergency
tonsillectomy is indicated because of
A. fever of more than 38.9 C (102 F)
B. obstructive tonsils associated with dysphagia
C. positive Streptococcus culture
D. swollen anterior cervical node
E. 3 attacks of sore throat in the past 1 year
Explanation:
The correct answer is B. Peritonsillar abscess (quinsy) is a localized area of abscess
that is typically unilateral and occurs in patients with tonsillitis. Group A beta-hemolytic
Streptococcus followed by anaerobes are the most common causative organisms.
Children usually present with sore throats which progress to peritonsillar abscesses.
Symptoms include severe throat pain with radiation to the ear, drooling from inability to
swallow saliva, trismus, and fever. Almost pathognomonic of the peritonsillar abscess is
a muffled, "hot potato" voice. On examination, there is unilateral swelling of the palate
and anterior pillar with the displacement of the tonsil downward and medially. The uvula
is moved away, off the fundi involved side. Treatment includes intravenous penicillin and
tonsillectomy. Although some series have documented good results using oral
antibiotics and needle drainage, it may need to be done many times. The major concern
in this patient, is the possibility of airway obstruction, although it's a rare event. Hence,
this patient should undergo an emergency tonsillectomy. Other indications for a
tonsillectomy are obstructive tonsils associated with sleep apnea, dysphagia, speech
defects, and failure to cry. Recurrent sore throats are relative indications for a
tonsillectomy. American Academy of Otolaryngology guidelines suggest that 7 sore

19

throats in 1 year or 5 sore throats in each of 2 years or 3 in each of 3 years, associated


with either fever of more than 38 C (100.4 F) or swollen anterior cervical nodes or
tonsillar exudate or positive strep culture are indications for tonsillectomy, not 3 sore
throats in 1 year (choice E).
Fever by itself is not an indication for tonsillectomy (choice A). Most of the time tonsillitis
is associated with sore throat and fever and this can be adequately treated with oral
antibiotics using penicillin. If the patient is allergic to penicillin, erythromycin oral
antibiotics can be used. If these fail, a first-generation cephalosporin may be used.
Swollen anterior cervical nodes (choice D) are not an indication for tonsillectomy by
itself. Swollen lymph nodes reflect the bacterial process from tonsillitis. These usually
respond to either oral or intravenous antibiotics, which will control tonsillitis.
Group A beta-hemolytic Streptococcus (choice C) is a common causative organism that
can be isolated from pharyngeal culture in children with tonsillitis. A positive culture by
itself, is not an indication for tonsillectomy, unless associated with the above-mentioned
complications.

A 68-year-old man comes to the emergency department because of a sudden loss of


vision in his left eye. He tells you that he underwent cataract surgery 6 days ago in his left
eye, and that he was promised that this would leave him with 20/20 vision. His vision is
now worse than it was before the surgery. Over the past few days he has been seeing
"floating objects" in his left field of vision, flashing lights, and at times it even seems as if a
curtain is coming down over the left eye. There is no pain associated with these
symptoms. His blood pressure is 120/80 mm Hg and pulse is 60/min. Ophthalmologic
examination of the left eye shows a blackish gray wavy material posteriorly. The right eye
is unremarkable. Physical examination is unremarkable. The most appropriate next step
is to
A. administer mannitol, intravenously
B. advise him to limit his physical activity
C. begin corticosteroids, intravenously
D. obtain immediate consultation with an ophthalmologist
E. schedule an immediate laser iridectomy
Explanation:
The correct answer is D. This patient has a retinal detachment, which is a known
complication of cataract surgery. This patient has the typical symptoms like flashing
lights, floaters, and "a curtain." This condition requires immediate evaluation by an
ophthalmologist and the treatment is to reattach the retina. Blindness is the feared
complication.
Mannitol (choice A) is given in acute glaucoma, not in a retinal detachment. Glaucoma
typically presents with severe pain and a red eye. It is associated with an increase of
intraocular pressure.
After you obtain consultation with an ophthalmologist, you should advise him to limit his
physical activity (choice B) to prevent further extension of the detachment.
If this patient had temporal arteritis it would be correct to begin corticosteroids,
intravenously (choice C). Temporal arteritis is associated with a unilateral headache, a
palpable, tortuous temporal artery, fatigue, myalgias, an elevated erythrocyte
sedimentation rate, and a visual loss. This patient's history and physical findings are
more consistent with retinal detachment than temporal arteritis.
A laser iridectomy (choice E) is part of the treatment for acute glaucoma, not a retinal

20

detachment. It is not usually performed as an emergent procedure during the acute


attack, it is often performed after the attack subsides.
A 72-year-old man is admitted to the hospital because of increasing left-sided ear pain,
low-grade fever, and purulent ear discharge. He treated himself with antibiotic eardrops
that he had at home and noticed some initial relief of symptoms, but for the past day the
pain has been getting worse and the purulent discharge from the left ear is increasing. He
also reports severe left-sided headaches early in the mornings. His medical history is
significant for diabetes mellitus, which is well controlled with insulin. He reports that he
has had previous left ear infections, which were treated with antibiotic eardrops and
occasional oral antibiotics. His temperature is 38.1 C (100.6 F), blood pressure is 140/76
mm Hg, and pulse is 84/min. There are no palpable lymph nodes on the neck or in the
supraclavicular region. There are no neurological deficits on his face. His right ear on
otoscopic examination is found to be normal. The left ear is tender on manipulation and
there is a purulent discharge coming from the external auditory meatus. On clearance of
the discharge from the left external auditory meatus, the canal is noted to be swollen and
the eardrum cannot be appreciated. There is no associated mastoid tenderness. The
most appropriate management is to
A. begin therapy with antibiotic eardrops
B. begin therapy with antibiotic eardrops and oral antibiotics
C. order a CT scan with contrast
D. provide therapy with intravenous antibiotics and watch for improvement
E. send a culture of the ear, irrigate with hydrogen peroxide, and provide acetic
acid drops
Explanation:
The correct answer is C. This gentleman has signs and symptoms of otitis externa. His
previous episodes were treated with antibiotic eardrops and oral antibiotics. Otitis
externa due to Pseudomonas in a diabetic patient can lead to malignant otitis externa.
Malignant otitis externa can invade the mastoid causing mastoiditis, and can result in
complications like meningitis and abscess. In this patient, the otitis externa, treated by
the patient himself with antibiotic eardrops, turned into malignant otitis externa.
Mastoiditis may be present and cannot be ruled out from mastoid non-tenderness. If this
invasive form of otitis externa is not diagnosed and appropriately treated with antibiotic
therapy, there is a potential for meningitis. Hence, a CT scan with contrast should be
performed to rule out any involvements of the mastoid. Once malignant otitis externa is
diagnosed and the extent of it is defined by means of clinical examination, an
otolaryngological examination and a CT scan with contrast, the patient should be
admitted for intravenous antibiotic therapy.
The patient is already taking antibiotic eardrops (choice A). These eardrops are not
enough to control and treat malignant otitis externa.
Administration of antibiotic eardrops and oral antibiotics against Pseudomonas, which is
the most common organism for malignant otitis externa, is essential once the diagnosis
is made (choice B). But most of the time, intravenous antibiotic therapy (choice D) is
essential in malignant otitis externa rather than oral antibiotics. Culture of the ears,
hydrogen peroxide irrigation, and acetic acid drops (choice E) are forms of the therapy
in otitis externa. But these measures, by themselves, would not treat malignant otitis

21

externa and a CT scan must be performed to rule out mastoid involvement.

A 3-year-old girl is brought to the clinic because of 3 days of fever and irritability. The
mother tells you that she thinks that she has another ear infection because she has been
pulling on her left ear for the past 2 days. You have been seeing this patient for well-child
examinations since she was born so you know that she is up-to-date on all immunizations
and she is rarely sick. She was treated for 2 episodes of otitis media, one at 1 1/2 years
of age and another at 2 1/2 years of age. Her temperature is 38.3 C (101.0 F). Physical
examination shows an erythematous and bulging left tympanic membrane, a loss of light
reflex, and decreased motility of the tympanic membrane. The right ear is
unremarkable.The most appropriate next step is to
A. obtain a consultation with a otolaryngologist for evaluation for tympanostomy
tubes
B. perform a tympanocentesis
C. prescribe a 1-month course of amoxicillin because she has recurrent otitis
media
D. prescribe a 7-10-day course of amoxicillin and schedule a follow-up visit in 2
weeks
E. reassure the mother that no treatment is necessary because this is most likely
due to a viral infection
Explanation:
The correct answer is D. This patient most likely has acute otitis media, which is most
likely due to an infection of the middle ear. It is often preceded by an upper respiratory
infection that leads to the accumulation of fluid in the middle ear that becomes infected
by viruses and bacteria. It is treated with amoxicillin for at least 7 days. Pain relief with
acetaminophen is also indicated. A follow-up visit should be scheduled in 2 weeks to
check if the otitis has resolved.
A consultation with an otolaryngologist for evaluation for tympanostomy tubes (choice A)
is inappropriate at this time. This is indicated if the patient has a persistent middle ear
effusions for more than 3 months that is unresponsive to antibiotic therapy.
A tympanocentesis (choice B) may be used in a child that is critically ill and fails to
respond to standard antimicrobial therapy. This patient should be treated with amoxicillin
at this time.
This patient most likely has acute otitis media and should be treated with a 7-10 day
course of antibiotics, not a 1-month course (choice C). Also, she does not have
recurrent otitis media, which is defined as 3 or more episodes of otitis media in 6
months or 4-5 episodes in 1 year. Individuals with recurrent otitis media are usually
given daily antibiotic prophylaxis with amoxicillin or sulfamethizole.
It is inappropriate to reassure the mother that no treatment is necessary because this is
most likely due to a viral infection (choice E) because this child most likely has acute
otitis media, which requires an antibiotic. It is often due to bacterial infections, however
viruses may be present in the middle ear fluid.

A 45-year-old woman who has recently moved into the area from out of state comes to
your office complaining of sneezing, itching, and watery eyes that she noticed when

22

spring arrived. She has never had them before. She also reports nasal congestion and
some clear nasal discharge, but denies fevers. Her past medical history is otherwise
remarkable for non-insulin dependent diabetes managed with glyburide. She takes no
other regular medications. The most appropriate diagnostic study is
A. a head CT scan
B. serum eosinophil levels
C. serum IgE levels
D. skin testing
E. there are no studies indicated at this time
Explanation:
The correct answer is E. This patient has allergic rhinitis (AR). AR may be seasonal or
perennial with significant overlap. For seasonal rhinitis there are well identified triggers
in spring and summer. Perennial is characterized by more chronic symptoms with
occasional flares. The diagnosis is solely based upon the history and physical
examination. Additional work-up is not cost effective and should be reserved for
refractory cases. The most common findings are sneezing, nasal pruritus, rhinorrhea,
postnasal drip, nasal discharge, eye watering, and pharyngeal itching.
A head CT scan (choice A) is useful in the diagnosis of sinusitis which is often infectious.
Both serum eosinophil levels (choice B) and serum IgE levels (choice C) are markers of
allergic disorders, but as with most such diseases the levels of these markers show a
poor correlation with disease presence or severity.
Skin testing (choice D) is a method of immediate hypersensitivity testing, the gold
standard of allergy diagnosis. Because such tests, although useful in identifying specific
allergens, are expensive and add marginal diagnostic yield when compared to physical
findings alone, they are not routinely recommended except in refractory cases.

You are examining the pupils of a 75-year-old man who was admitted to the hospital
because of unstable angina. When examining his right pupil you note that it reacts
normally to light. Examination of the left pupil also reveals normal reaction to light. When
you shine the light quickly back to the right eye, the right pupil dilates. A similar test on the
left side yields constriction of the left pupil. Your clinical examination findings are
consistent with
A. astrocytoma involving the right optic tract, visual acuity 20/30
B. mature cataract in the right eye, visual acuity light perception
C. optic nerve sheath meningioma, visual acuity 20/400
D. physiologic anisocoria
E. right posterior cerebral artery CVA with right homonymous hemianopsia, visual
acuity 20/40
Explanation:
The correct answer is C. The pupillary exam reveals an afferent pupillary defect (APD)
or Marcus-Gunn pupillary defect. This is an important clinical sign that is always
indicative of pathology. An APD is present when the central pupillary regulatory centers
receive unequal input from the optic nerves. Thus, with a swinging flashlight test, the
affected optic nerve "sees" less light and the pupil dilates. Any optic nerve injury such as
optic neuritis, ischemia, or compression will cause an APD. In this case, a meningioma
is causing compression of the optic nerve. The presence of an APD that otherwise has

23

not been explained is an indication for an ophthalmology consultation.


Optic tract lesions are a cause of APDs, but they are seen on the contralateral side. This
is because more nasal (contralateral) retinal fibers desiccate at the optic chiasm than
there are temporal (ipsilateral) retinal fibers (choice A).
Media opacities such as cataracts (choice B), corneal disease, or vitreous hemorrhage
are not a cause of an APD.
Physiologic anisocoria (choice D) is common. It denotes unequal pupils, but it is a
diagnosis of exclusion. Furthermore, pupillary responses are normal and an APD would
be absent in physiologic anisocoria.
The retinal ganglion cells synapse in the lateral geniculate bodies (LGB). Fibers in the
optic tract that regulate pupillary function exit the optic tract anterior to the LGB. Thus,
lesions posterior to the LGB do not cause pupillary abnormalities. Therefore, cerebral
vascular accidents (choice E) involving the posterior circulation do not cause APDs.

A 42-year-old man comes to the emergency department in which you are working with a
chief complaint of progressive blindness. He states he was playing golf with his daughter
when he noticed difficulty with his vision. He states that he "looked into the sun" to follow
his golf shot and that, "the vision never returned." His past medical history is remarkable
for osteoarthritis of his hip for which he takes celecoxib. He is otherwise healthy. Review
of systems elicits mild headache, joint pains, and nausea. His temperature is 38.3 C
(101.0 F), blood pressure is 168/90 mm Hg, and pulse is 88/min. He has a visual acuity of
hand motions (HM) in his right eye and counting fingers in his left eye. He is upset and
tearful at times. You note that the triage nurse charted his visual acuity at 20/200 in both
eyes. Pupils are 6 mm in both eyes with poor reaction to light and a trace afferent
pupillary defect in the right. Tonopen eye pressures are normal in both eyes. Extraocular
movements are full in both eyes. Confrontational visual field testing shows a temporal
visual field defect in the right eye and a temporal visual field defect in the left eye. Slit
lamp examination and dilated direct ophthalmoscopy is normal in both eyes. The best
next step in the management of this patient is to
A. determine blood alcohol level and do urine toxicology screening
B. get an immediate ophthalmology consultation
C. investigate possible secondary gain in regard to blindness
D. order STAT neuroimaging and neurosurgical consultation
E. perform a lumbar puncture
Explanation:
The correct answer is D. This patient has the classic findings of pituitary compression of
the optic chiasm. The acuteness of his presentation suggests that he has pituitary
apoplexy or hemorrhage and sudden enlargement of a pituitary adenoma. The most
important clinical symptoms/signs are sudden loss of vision (in this patient even
between the triage nurse and physician exam), abnormal pupils, headache with nausea,
and a bitemporal visual field deficit. Urgent neuroimaging and neurosurgical consultation
is indicated.
A positive blood alcohol or toxicology screen (choice A) would not explain the important
clinical signs of an afferent pupillary defect and bitemporal hemianopsia.
Ophthalmology consultation (choice B) is unnecessary at this point and delays
appropriate triage of this patient.
Secondary gain (choice C) is a common concern when patients complain of sudden
visual loss, but it is important to realize that in this case there are worrisome clinical

24

signs that are consistent with his presentation. The saying of "pupils never lie" means
that an afferent pupillary defect is always indicative of pathology.
A lumbar puncture (choice E) could be a potentially dangerous procedure in this context
of intracranial hemorrhage and increased intracranial pressure. Neuroimaging should be
performed first prior to lumbar puncture in patients where increased intracranial
pressure is a possibility.

A 53-year-old man who has been taking amoxicillin-clavulanate for 10 days for chronic
sinusitis is brought to the emergency department by his wife because of the development
of increasing fever and a red, swollen right eye over the past 12 hours. His temperature is
38.7 C (101.6 F). Physical examination shows periorbital edema and erythema,
conjunctival injection, chemosis, and proptosis. Cranial nerve examination and
extraocular movements are normal. Nasal examination shows purulent material in the
superior meatus and a diffusely congested mucosa. The most likely explanation for these
new symptoms is
A. an allergic reaction to amoxicillin-clavulanate
B. contiguous spread of infection from the ethmoid sinus through the lamina
papyracea
C. contiguous spread of inflammation or infection from the sinuses to the
meninges
D. development of a frontal subperiosteal abscess
E. development of a tumor in the cavernous sinus
Explanation:
The correct answer is B. This patient most likely has developed orbital cellulitis, which is
one of the most common complications of sinusitis. Orbital cellulitis develops by direct
spread of infection from the ethmoid sinus through the lamina papyracea (the very thin
bone that separates the orbit from the ethmoid sinus). A CT scan or MRI should be
obtained and the treatment consists of intravenous antibiotics and possibly surgical
drainage of the ethmoid sinus. Blindness and meningitis may occur if this is not treated
aggressively.
An allergic reaction to amoxicillin-clavulanate (choice A) is an unlikely explanation for his
symptoms because of the lack of a rash and the presence of such localized symptoms
(one eye).
It is unlikely that this patient's symptoms are caused by the direct spread of
inflammation or infection from the sinuses to the meninges (choice C) because he does
not have a headache or a stiff neck. This patient's symptoms are more consistent with
orbital cellulitis than meningitis, but both are complications of sinusitis.
The development of a frontal subperiosteal abscess (choice D), which is a complication
of frontal sinusitis, typically presents with a tender, "doughy" swelling on the forehead. It
is also called Pott's puffy tumor. This diagnosis is inconsistent with this patient's
presentation.
The development of a tumor in the cavernous sinus (choice E) is an unlikely explanation
for this patient's symptoms because it is usually associated with ophthalmoplegia and 3rd
cranial nerve abnormalities. Also, it is extremely unlikely that this patient would have
developed a tumor so quickly.

25

A 92-year-old man is brought to the clinic by his son and daughter who tell you that their
father "has gone blind". Further investigation uncovers that his visual loss has been a
slowly progressive process with no ocular pain. His son and daughter are concerned
because the patient lives alone. The patient does not complain of any difficulty seeing
and is agitated that he has been brought to your office. Examination reveals normal vital
signs and a visual acuity of 20/400 in both eyes. Pupil examination, extraocular
movements, and confrontational visual fields are all normal. A penlight exam of the eye
shows a yellow-brown color to the lens in both eyes, but is otherwise unremarkable.
Direct ophthalmoscopy is very difficult and a sharp view of the retina is not possible. The
family should be advised that:
A. Age alone is a common cause of decreased vision and his level of vision is
what you would expect for a patient his age
B. Eye surgery is necessary to prevent total blindness (no light perception)
C. Glaucoma is the most likely diagnosis and prompt ophthalmology consult is
indicated
D. He has macular degeneration and low vision aides should be considered to
assist with activities of daily living
E. Tell the patient and family that as long as the patient is comfortable with his
vision no treatment is necessary
Explanation:
The correct answer is E. This patient has cataracts in both eyes. The typical clinical
presentation of a patient with cataracts is that of slowly progressive visual loss. Patients
may also complain of difficulty with glare and bright lights. They often note decreased
color brightness and contrast sensitivity. On exam, a lens that has a cataract will appear
yellow-green to yellow-brown. A mature cataract will present as a white lens and visual
acuity of hand motions or light perception. With advanced cataracts, the fundoscopic
exam may be difficult or not possible. The treatment for cataracts depends on the level
of visual deficit experienced by the patient. Appropriate treatments range from
observation/education, spectacles, and cataract surgery. If a patient with cataracts is
comfortable with his current vision, then treatment with glasses or surgery is not
necessary. Referral to an ophthalmologist may be appropriate for more in depth
counseling based on the level of concern of the patient.
The level of visual acuity (choice A) should not be expected to decrease with age. Age
alone is not an appropriate reason for decreased vision. A patient with healthy eyes and
visual pathways should expect to see 20/20 regardless of age.
Although cataract surgery (choice B) is very successful in restoring vision, it is not a
necessary procedure to prevent blindness. Cataracts alone do not cause a loss of light
perception. It is appropriate to educate patients that they have cataracts and that a
surgery exists that may help them see better. However, cataract surgery remains an
elective procedure.
Patients with glaucoma (choice C) do not complain of decreased visual acuity. They
may have decreased peripheral vision or scotomata, but even in advanced cases of
glaucoma the visual acuity may remain 20/20.
Macular degeneration (choice D) is a common cause of decreased vision in the elderly.
With this case, the macula could not be viewed, suggesting a media opacity as the
cause of visual impairment.

26

An 11-year-old girl is brought to your office by her father. She was sent home from school
today and told to be seen by a physician. Her father reports that her left eye has been red
for the last 2 days and this morning her right eye began appearing pink as well. He also
noted a moderate amount of watery discharge from both eyes. She complains of a
burning sensation and feels like she needs to blink often. Her temperature is 37.0 C (98.6
F). Ocular movements are intact and both pupils are equal, round, and reactive. Diffuse
hyperemia is noted of both eyes. Fluorescein staining is negative. The remainder of the
exam is within normal limits. The additional finding that should prompt immediate referral
to an ophthalmologist is
A. concurrent upper respiratory symptoms
B. decreased visual acuity
C. history of severe asthma
D. history of varicella infection 1 year ago
E. severe pruritus in both eyes
F. yellow-green discharge from the eye
Explanation:
The correct answer is B. This patient is presenting with symptoms consistent with acute
conjunctivitis. The most common causes are infectious (viral and bacterial), allergic, and
chemical. Treatment usually consists of comfort measures such as a cool compress and
topical antibiotics or ocular allergy medications if indicated. One of the most important
considerations in evaluating patients with conjunctivitis is to rule out any visionthreatening conditions such as iritis, keratitis, glaucoma, or a corneal ulceration.
Symptoms such as marked photophobia, decreased visual acuity, or globe pain suggest
that ocular structures other than the conjunctiva are involved and should trigger
immediate ophthalmologic evaluation.
Upper respiratory symptoms (choice A) are often found in association with viral
conjunctivitis.
A history of asthma, (choice C) and pruritus (choice E) suggest allergic conjunctivitis as
the etiology of this patient's symptoms. Other associated allergic disorders, such as
atopic dermatitis or allergic rhinitis, would also support this diagnosis.
A history of varicella infection (choice D) without any current lesions is not significant in
this case. However, if there were symptoms suggestive of active Herpes simplex or
Varicella zoster infection, ophthalmologic referral would be appropriate.
A yellow-green discharge (choice F) may raise suspicion of bacterial infection and
prompt the addition of topical antibiotics to the therapeutic regimen. In addition, a culture
of the discharge may be sent. However, ophthalmologic evaluation is not necessary.

A 24-year-old woman underwent a laparoscopic appendectomy for acute appendicitis.


She was given 3 doses of intravenous antibiotics and was to be discharged on the
second postoperative day. On the morning of second postoperative day, while trying to
get out of the bed, she slipped, fell, and bumped her nose on the bedside table. You are
called to the floor to evaluate the patient because she is now complaining of some
difficulty in breathing through her nose. On examination, the patient is afebrile, and is
feeling well except for the nasal trauma. You notice a hyponasal noise during the
conversation. Examination of the nose reveals a swollen nose with a bruised outside and
a small laceration on the nasal ala. Examination of the nasal cavity reveals a swelling in
the region of the nasal septum, which is more prominent to one side. Examination of this

27

swelling by means of a nasal speculum, reveals that it is soft and fluctuant. The nasal
trauma happened 1 hour ago. At the end of the examination, the patient says that apart
from hyponasal voice and some amount of pain, she is not feeling any discomfort and is
ready to go home. You advise this patient that
A. continued observation in the hospital is necessary
B. intravenous antibiotic therapy is indicated
C. oral antibiotic therapy for 1 week is necessary
D. she can be discharged home and can follow-up in the office in 2 weeks
E. surgical drainage of the swelling is indicated at this time
Explanation:
The correct answer is E. This patient has a septal hematoma from the nasal trauma. A
septal hematoma is a collection of blood between the nasal mucosa and the
cartilaginous septum. It can develop after nasal trauma (including nasal surgery) and
can be unilateral, but is more commonly bilateral. Patients typically complain about
difficulty in breathing or may sound hyponasal. If the hematoma is left untreated, it might
become infected, resulting in increasing pain. Examination of the nose shows a swelling
in the region of the septum. Pressing on this swelling with a nasal speculum reveals a
soft and fluctuant mass, (contrary to a hard and non-fluctuating mass as in a deviated
septum or preexisting deformity). The cartilage of the nasal septum receives its
nourishment from the overlying mucosa. The hematoma blocks this supply. If the
hematoma is not cleared quickly, the cartilage can become ischemic and ultimately
necrotic. The necrotic tissue can slough, eventually resulting in a septal perforation or a
saddle nose deformity. Both of these deformities are almost impossible to fix. Once a
septal hematoma is identified in the nose, the patient should be advised for adequate
drainage. The nasal septum should be anesthetized either topically and/or by means of
injection. A large spinous needle should be introduced at a dependent area to evacuate
the hematoma. Ideally, this opening is enlarged with a #11 blade. A Penrose drain can
be left through this incision to keep this opening open for a day or two. Then, a bilateral,
tight anterior nasal pack needs to be placed to re-appose the septal mucosa. The
patient needs to be monitored for re-accumulation of the septal hematoma for one or
two days.
Continuing to observe the patient in the hospital (choice A) is not a good option. For the
reasons explained above, an increase in the hematoma size within the nasal septum
might compromise the blood supply to the cartilage, which can become ischemic and
necrotic. To prevent this, early drainage of the septal hematoma is advised.
Intravenous antibiotics (choice B) are not indicated in a non-infected septal hematoma.
Oral antibiotics are not indicated in a clean nasal septal hematoma (choice C). A
hematoma, when adequately and appropriately drained early in the process, will resolve
spontaneously and no antibiotic therapy will be needed.
The patient should not be discharged (choice D), as the septal hematoma can worsen
causing increased pain and cartilage necrosis. Even when adequately drained, this
patient needs to be followed up within a few days in the office to examine for any reaccumulation of the nasal septal hematoma.

A 70-year-old woman comes to the emergency department with a 2-day


history of a right-sided facial rash and right eye pain. She has no significant
medical history and is on no medications. She is unaware of her childhood
diseases and prior immunizations. She states that her husband recently had

28

"the flu and a bout of pink eye." Physical examination shows a vesicular rash
on her right scalp and forehead, right upper eyelid, right side of the nose, and
the tip of the nose. The visual acuity of the right eye is 20/50 and the left eye is
20/20. There is no relative afferent papillary defect. The intraocular pressure is
15mm Hg in both eyes. The conjunctiva is diffusely red and injected, and slit
lamp examination reveals multiple small epithelial dendrites on the cornea of
the right eye. The most appropriate next step in management is to
A. obtain herpes simplex antigen detection studies
B. prescribe oral acyclovir
C. prescribe oral antibacterial therapy
D. prescribe topical acyclovir cream for the skin and topical acyclovir
drops for the eye
E. prescribe topical antibiotic cream for the skin and a topical antibiotic
ointment for the eye
Explanation:
The correct answer is B. The dermatomal distribution of a vesicular rash
should lead you to the diagnosis of herpes zoster. Herpes zoster
ophthalmicus is zoster in the V1 distribution presenting with ocular
involvement. Oral acyclovir therapy was found in randomized clinical trials to
reduce viral shedding from vesicular skin lesions, decrease systemic
dissemination of the virus, and ameliorate the incidence and severity of the
most common ocular complications (dendritic keratitis, uveitis, and stromal
keratitis).
Herpes simplex antigen detection studies (choice A) would not be helpful for
herpes zoster.
Oral antibacterial therapy (choice C) would be inappropriate for the
management of herpes zoster, which is a viral infection.
Topical antiviral drops (choice D) have not been found to be effective for the
treatment of herpes zoster ophthalmicus.
A topical antibiotic (choice E) may be used on the skin or in the eye to
prevent secondary bacterial infection, but this is not the most appropriate
next step in management.

29

A 47-year-old male is brought to the emergency department after he is injured in a fist


fight. He was punched in the face multiple times and has pain and swelling around his left
eye. Physical examination demonstrates ecchymosis and swelling of his left lower eyelid.
There is a mild left periorbital swelling but no obvious tenderness or step off deformity on
palpation. The cornea, lens, and anterior chamber are clear bilaterally. The pupils are
equal and reactive. There is a mild restriction of upward gaze in his left eye, but there is
normal abduction and adduction. Extraocular movements are normal in the right eye.
Sensation in the distribution of the infraorbital nerve is intact. A coronal CT scan of the
orbits is shown . The most likely complication of this type of orbital injury is
A. blindness
B. entrapment of the inferior rectus muscle
C. entrapment of the medial rectus muscle
D. an orbital pseudotumor
E. sinusitis
Explanation:
The correct answer is B. The coronal CT scan of the orbits clearly demonstrates an
orbital floor fracture. The patient's exam finding of restricted upward gaze also suggests
injury to the inferior rectus muscle. Orbital wall fractures most often occur in the orbital
floor and sometimes in the medial wall of the orbit, because these are the weakest
regions of the bony orbit. The proximity of the paranasal sinuses, nerves, vessels,
extraocular muscles, globe, and other orbital structures predispose them to a wide
variety of possible damage from injuries producing orbital fractures. The most common
extraocular muscle entrapped in orbital floor fractures is the inferior rectus muscle,
which limits upward gaze.
Blindness (choice A) is not a complication of orbital floor fractures. Blindness can occur
if there is direct injury to the globe itself or if there is injury to the optic nerve. There are
no signs of globe or optic nerve injury in this patient's physical examination or in the CT

30

scan.
Entrapment of the medial rectus muscle (choice C) is a complication of fractures of the
medial orbital wall. Entrapment of this extraocular muscle is manifested by limited
abduction of the eye.
An orbital pseudotumor (choice D) is not associated with orbital injury. It is an idiopathic
granulomatous inflammation of the orbit that often presents with chemosis, proptosis,
and pain. The most common structure in the orbit to be involved is the lacrimal duct.
Sinusitis (choice E) is not associated with orbital injury. Sinusitis is often due to
underlying allergies. Orbital floor fractures or other fractures of the orbit do not
predispose patients to the development of sinus disease.

A 19-year-old premedical student comes to the student health center complaining of a 48hour history of fever, chills, a worsening sore throat with pain on swallowing and a
headache. He has no other medical history and takes no medications. He is sexually
active with one female partner, denies homosexual encounters and injection drug use. He
appears mildly ill. Physical examination shows a markedly erythematous pharynx and
tender anterior cervical lymphadenopathy. There is no hepatosplenomegaly present. The
most appropriate next step is to order
A. HIV serum RNA levels
B. a Monospot test
C. a rapid strep test
D. a throat culture on Thayer-Martin medium
E. a throat swab and culture
Explanation:
The correct answer is C. This patient most likely has pharyngitis caused by the group A
beta-hemolytic streptococcus. His signs and symptoms are classic for presentation with
an infection caused by this bacteria. This is the only form of acute pharyngitis for which
antibiotics are definitively indicated. Antibiotic treatment prevents well-defined
suppurative complications such as peritonsillar abscess and nonsuppurative
complications such as rheumatic fever. Although the gold standard diagnostic test is the
throat swab and culture (choice E), this test is only useful when follow up is available. In
a college student, the rapid strep test, which is an ELISA test looking for the bacterial
antigen is the most practical test because the results are available within 1 hour.
Since this patient has no risk factors for HIV, testing for HIV serum RNA levels (choice
A) is not indicated.
The Monospot test (choice B) is a test for heterophile antibody in blood, which indicates
infection with Epstein-Barr virus (infectious mononucleosis). This typically presents with
a few-week history of fatigue, malaise, and a sore throat. Examination often shows
pharyngeal edema, erythema, and palatal petechiae, lymphadenopathy, and
splenomegaly. Hepatomegaly may also occur.
A throat culture on Thayer-Martin medium (choice D) would be used to test for
gonococcal disease, which this patient is not suspected of having.

A 12-year-old girl is brought to the office by her mother with complaints of foul smelling
discharge, and hearing loss in left ear for the past few days. The girl has no past medical
history. She underwent an uneventful appendectomy at the age of 9, after which she was

31

discharged from the hospital within 3 days. On examination, the girl is afebrile with
normal vital signs. Head and neck examination did not reveal any abnormalities.
Examination of the left ear shows purulent discharge in the external auditory canal. After
clearing the external auditory canals, otoscopic examination shows a perforation in the
pars flaccida. Apart from the perforation, small bits of amorphous white debris are also
found in the left ear. Examination of the right ear is normal. Appropriate management of
this patient's condition is
A. antibiotic eardrops, decongestants, and follow-up in 4 weeks
B. mastoidectomy
C. oral antibiotics and follow-up in 2 weeks
D. reassurance and follow-up in 3 weeks
E. tympanoplasty
Explanation:
The correct answer is B. The condition described in this girl is a cholesteatoma. A
cholesteatoma occurs when the middle ear is lined with stratified squamous epithelium.
The squamous epithelium desquamates in a closed space, which cannot be cleared and
hence accumulates serving as a culture medium for the organisms. Cholesteatomas
have the ability to destroy bone, including ossicles. Those arising in association with a
perforation in the pars flaccida are called primary acquired cholesteatoma, while those
arising in association with marginal perforations are called secondary acquired
cholesteatoma. The presence of cholesteatoma greatly increases the probability of the
development of serious complications, such as purulent labyrinthitis, facial paralysis, or
intracranial suppurations like meningitis, brain abscess, subdural empyema, or epidural
abscess. They are usually recognized by the small bits of amorphous white debris in the
middle ear, and by the destruction of the bone of the external auditory canal superior to
the perforation. A CT scan of the temporal bone is helpful in determining destruction of
the bone. Cholesteatoma requires surgical treatment. The primary goal of the operation
is to make the ear safe and the secondary goal is to maintain, or improve the hearing.
The objective of the therapy is to remove the cholesteatoma or exteriorize it. A
mastoidectomy, sparing the tympanic membrane, is the appropriate form of therapy for
a cholesteatoma. In a radical mastoidectomy, the middle ear including the attic, the
antrum, and the mastoid antrum are converted into one cavity that is in communication
to the exterior, through the ear canal. The modified radical mastoidectomy spares the
tympanic membrane remnants and ossicles to preserve the hearing.
Antibiotic eardrops and decongestants are not sufficient forms of therapy in treating a
cholesteatoma (choice A). Delay in the treatment without a mastoidectomy, usually
results in the progression of the disease.
Oral antibiotics (choice C) are not a sufficient form of therapy in treating a
cholesteatoma. Although after a mastoidectomy, this condition can be treated with
antibiotics, initially surgical treatment is the option.
Reassurance and follow-up in 3 weeks to see if the perforation closes spontaneously
(choice D) is not optimal treatment. Cholesteatoma usually progresses, with the
destruction of the bone and becomes a culture medium for the bacteria, unless a
surgical debridement is carried out.
Tympanoplasty (choice E) is not a safe option, as this will not treat the primary

32

pathology in a cholesteatoma.

A 12-month-old girl is brought to the office for a well-child appointment. She is in the 75th
percentile for both weight and height. Examination is normal except for her ocular
alignment. You note an esotropia of the right eye. When the left eye is covered you note
normal alignment of the right eye. When the left eye is uncovered it shows an esotropia of
the same degree. The child has a full range of motion of her extraocular movements. The
mother states that she has had a "lazy eye" since birth. At this time the most correct
statement about this patient's condition is:
A. All patients with strabismus should be evaluated at some point with a CT scan
of the head and brain
B. Amblyopia from strabismus is reversible if identified and treated appropriately
C. Any loss of visual acuity that is present now will be permanent
D. Most children will outgrow strabismus as their visual system matures
E. Right 4th cranial nerve palsy cannot be ruled out in this patient
Explanation:
The correct answer is B. Strabismus is an important diagnosis to make. Resultant
amblyopia is treatable and reversible if the child is referred to an ophthalmologist
promptly. Treatment with spectacles or patching is very effective in the first 5 years of
life, and often later depending on the degree of amblyopia and its cause.
Neuroimaging is considered for selected cases of atypical ocular misalignment or with
any suspicion of cranial nerve palsies, but routine imaging for all children with
strabismus is not necessary based on ocular findings alone (choice A).
Visual acuity loss from amblyopia will improve with treatment and so any loss of visual
acuity that is present now will not be permanent (choice C) is incorrect. However, stereo
vision loss is harder to treat. Many patients with amblyopia will note subtle subjective
visual loss in their amblyopic eyes that may not show up on visual acuity measurements
Children do not grow out of strabismus and you should never wait for the child to
"outgrow" ocular misalignment (choice D). Delay in referral to an ophthalmologist will
make the treatment of amblyopia longer and more difficult.
This patient does not have a 4th cranial nerve palsy (choice E). The patient has full
extraocular movements and a forth cranial nerve palsy will produce an exotropia
(outward deviation) and hypertropia (upward deviation) of the affected eye.

A 4-year-old girl is brought to the clinic because of a 2-week history of cough and severe
sore throat. Her temperature is 38.1 C (100.6 F), blood pressure is 90/50 mm Hg, pulse is
100/min, and respirations are 18/min. Physical examination reveals a well-developed girl
with mild stridor. There is submandibular, submental, anterior and posterior cervical and
jugulodigastric lymphadenopathy. The throat is erythematous, but there are no exudates.
The remainder of the examination is unremarkable. A chest radiograph shows air in the
superior mediastinum and clear lungs. After drawing blood and sending it for a complete
blood count, the next step in evaluating this patient is to
A. obtain a throat culture and a neck x-ray
B. order a CT of chest
C. order a CT of chest and a neck x-ray

33

D. order a CT of chest and a throat culture


E. perform direct laryngoscopy and a throat culture
Explanation:
The correct answer is A. This patient most likely has a retropharyngeal abscess. The
next step in evaluating this is a lateral radiograph of the neck. This will also allow for
assessment of a foreign body in the epiglottis. A throat culture is necessary to guide
antibiotic therapy. A retropharyngeal abscess is a disease of children under the age of 5
years. Tissue in the back of the throat of young children allow a purulent collection to
form. However, this does not happen in adults, as the potential space does not exist.
Clinical signs of a retropharyngeal abscess include stridor, severe sore throat, difficulty
swallowing, and drooling. There will be an elevated white blood cell count. On plain film
there will be widening of the retropharyngeal soft tissues with a possible impingement
on the airway. The abscess can dissect into the mediastinum causing mediastinitis.
Further complications include pneumonia and osteomyelitis. The treatment involves
intravenous antibiotics, possible airway protection and surgical drainage.
A CT of the chest (choice B) will not detect the retropharyngeal abscess in the neck.
A CT of the chest and a neck x-ray (choice C) is a complete radiographic evaluation for
retropharyngeal abscess with mediastinitis. A throat culture, however, is necessary to
direct antibiotic therapy in this case. Broad spectrum antibiotics should be fine tuned
with cultures whenever possible.
A CT of the chest and a throat culture (choice D) will not detect the retropharyngeal
abscess in the neck. The pneumomediastinum is a secondary finding of retropharyngeal
abscess. A throat culture is indeed necessary to direct antibiotic therapy in this case.
A direct laryngoscopy and a throat culture (choice E) would not be the way to approach
this scenario. The next step in evaluating this is a lateral radiograph of the neck. This
will also allow for assessment of a foreign body in the epiglottis and an evaluation for a
retropharyngeal abscess.

A 67-year-old woman comes to the office because of "ringing in her ears," a feeling of
"spinning", and a progressive loss of hearing in her right ear over the past 5 months. She
says that this all began "a while ago with a slight feeling of unsteadiness." She never
went to the doctor because she thought she was "going crazy," but now her husband is
getting worried because the television needs to be much louder and she constantly says
"what?" when he speaks to her on her right side. She has no chronic medical conditions,
does not take any medications, and does not drink alcohol. Examination shows
nystagmus, but no other abnormalities. The most likely diagnosis is
A. benign positional vertigo
B. benign recurrent vertigo
C. Meniere's disease
D. toxic labyrinthitis
E. vestibular neuronitis
Explanation:
The correct answer is C. This patient has Meniere's disease, which is characterized by
tinnitus, vertigo, and progressive hearing loss. It is thought to be related to a
degeneration of the vestibular and cochlear hair cells. The treatment includes bed rest,
a low-salt diet, dimenhydrinate, cyclizine or meclizine.
Benign positional vertigo (choice A) is characterized by paroxysmal vertigo and

34

nystagmus. It is brought on by certain changes in position. Hearing loss is not present.


The cause is idiopathic.
Benign recurrent vertigo (choice B) and vestibular neuronitis (choice E) are two names
for the same syndrome, which is characterized by the sudden onset of vertigo, nausea,
and vomiting. There is no change in hearing.
Toxic labyrinthitis (choice D) presents with vertigo. It is due to medications and alcohol.
Aminoglycosides are among the most common culprits. This patient does not take any
medications and denies any alcohol.

A 26-year-old medical student comes to the office because of a 1-month history of an


excruciating left-sided headache. She says that the pain is almost constant and that there
is nothing that exacerbates or relieves it. She has had headaches in the past but "nothing
like this." She tells you that she is not exactly sure what a brain tumor feels like, but she
would not be surprised if this is it. She does not have any other symptoms, takes no
medications, does not smoke cigarettes, and rarely drinks alcohol. Her temperature is
36.7 C (98.0 F), blood pressure is 110/80 mm Hg, pulse is 60/min, and respirations are
14/min. A complete physical examination, including a neurologic and funduscopic
examination, is unremarkable. Against your better judgement, you decide to order an MRI
of the head and a lumbar puncture, in which of course, you do not find any abnormalities.
You prescribe her lithium, ergotamine, sumatriptan, propranolol, and amitriptyline. She
returns one month later and complains that none of these medications were helpful and
that the pain has gotten so bad that she had one of her "resident friends" prescribe
narcotic agents. At this time you should
A. advise her to see a psychiatrist
B. begin immediate corticosteroids
C. refer her to a dentist
D. refer her to a pain management specialist
E. take a temporal artery biopsy
Explanation:
The correct answer is C. The approach to the patient with a headache is difficult
because often, at times the complete evaluation is normal. A physical examination,
including a neurologic and funduscopic exam, should almost always be performed.
Neuroimaging is generally not indicated if the physical exam is normal. Chronic pain can
be due to typical headaches, such as tension headaches and cluster headaches, a
brain tumor, temporal arteritis, pseudotumor cerebri, or other miscellaneous causes like
disorders of the teeth, jaws, or related structures. Dental caries that progress to infection
of the dental pulp, the alveolar bone, and the soft tissues may cause a severe unilateral
headache that can be confused with any of the above causes. Before you do "a million
dollar" work-up for a headache with a completely normal physical examination,
especially when the usual migraine, tension, and cluster headache treatments are not
helpful, a trip to the dentist may easily solve the problem.
While depression has been associated with chronic headaches, it seems that this
patient should go to the dentist for an evaluation before you advise her to see a
psychiatrist (choice A). You should first try to seek an organic cause of disease before
you blame the chronic pain on depression.
Immediate corticosteroids (choice B) would be appropriate if you suspected temporal
arteritis with ocular complications. She would probably have a tender temporal artery,
scalp pain, and other systemic symptoms such as malaise, morning stiffness, shoulder,

35

and hip pain. This patient only complains of a headache.


Chronic pain may be managed by a pain management specialist (choice D). However,
this patient should first see a dentist before you recommend this treatment. You must
understand that dental pain commonly causes a headache.
A temporal artery biopsy (choice E) is inappropriate at this time because it is very
possible that this patient's pain comes from the teeth or jaw. Temporal arteritis typically
affects elderly individuals and is associated with a tender temporal artery, ocular, and
systemic symptoms. This patient does not have any of these symptoms.

A 37-year-old physician comes to the office with a chief complaint of diplopia. She noted
these symptoms 2 days ago and feels that things are "getting worse and that its giving
me a headache." Her diplopia is worse when she looks up and when she looks to the left.
She also relates difficulty in keeping her right eye open. Her past medical history is
remarkable for mild hypertension, "reflux", and uterine fibroids. Her temperature is 37.2 C
(99.0 F), blood pressure is 138/76 mm Hg, and pulse is 74/min. Her visual acuity is 20/20
in both eyes. Pupil examination reveals an 8-mm pupil on the right that does not react to
light and a 5-mm pupil on the left that reacts to 3-mm with light. Her extraocular
movements are limited in the right eye. She has difficulty adducting her right eye and
elevating her right eye. The left eye has normal extraocular movements. The rest of her
ocular examination, including direct ophthalmoscopy, is normal in both eyes. The most
appropriate next step is to
A. begin therapy with zolmitriptan
B. evaluate the patient for diabetes mellitus
C. optimize control of the patient's hypertension and refer the patient to a
neurologist
D. order an urgent MRI/MRA of the brain
E. refer the patient to an ophthalmologist
Explanation:
The correct answer is D. This patient has an acute third cranial nerve palsy. Patients
with third nerve palsies will have extraocular muscle abnormalities, a dilated pupil, and
ptosis. The eye is typically deviated "down and out" due to the unopposed actions of the
lateral rectus and superior oblique muscles. Some patients do not have diplopia simply
because their ptosis is pronounced enough to occlude the vision out of the paretic eye.
The pupil exam is critical in the evaluation of a third nerve palsy. Patients with a dilated
pupil have a compressive lesion on the third nerve until proven otherwise. The third
nerve passes between the posterior cerebral artery and the superior cerebellar artery,
and then parallels the posterior communicating artery. An aneurysm of any of these
arteries (most commonly the posterior communicating artery), can cause a compressive
lesion of the third nerve. An emergent MRI/MRA is the best test to evaluate compression
of the third nerve, as it shows aneurysms greater than 4 mm as well as subtle neoplastic
or inflammatory processes that could be causing compression.
A migraine headache (choice A) would be an unlikely cause of this presentation. The
best first step is neuroimaging, and if this is negative then other causes such as
migraines could be considered.
A "diabetic" third nerve palsy (choice B) presents as a "pupil-sparing" third nerve palsy.
The etiology is presumably microvascular infarction of the third nerve. The fibers of the
third nerve that control pupillary responses run on the outside of the nerve and thus are
less likely to be affected by microvascular ischemia. This anatomic relationship also

36

explains why compressive lesions of the third nerve will almost always show pupillary
deficits.
Control of the patient's hypertension (choice C) is not of primary concern and referral to
a neurologist will delay appropriate neuroimaging.
Referral to an ophthalmologist (choice E) will also delay appropriate triage of this
patient. Later, after the etiology of the third nerve palsy has been addressed,
ophthalmology referral may be helpful in dealing with issues of diplopia.

A 10- year-old girl is brought to the clinic because of a 2-day history of a sore throat and
fever. The mother reports that the fever has been as high as 39 C (102.2 F) and that the
child is complaining of pain on swallowing. She has had no rhinorrhea, cough, or ear
pain, and no one else is ill at home. Physical examination reveals a well-appearing girl
with an exudative pharyngitis and multiple 1-2-cm tender submandibular lymph nodes.
There is no other adenopathy, rash, or hepatosplenomegaly present. A rapid antigen
detection test for group A streptococcus done on a swab of the posterior pharynx is
positive. The most appropriate action at this time is to
A. perform a complete blood count and monospot test
B. repeat the rapid antigen detection test
C. send a throat swab for culture
D. treat with oral amoxicillin-clavulanate
E. treat with oral penicillin
Explanation:
The correct answer is E. The positive rapid test for group A streptococcus (S. pyogenes)
confirms the diagnosis of streptococcal pharyngitis, and antibiotic therapy is essential
for the prevention of suppurative complications (e.g., adenitis, peritonsillar abscess) as
well as rheumatic fever. Penicillin remains the treatment of choice for streptococcal
pharyngitis in non-allergic patients, as treatment failures are extremely rare.
Infectious mononucleosis is another potential cause of fever, pharyngitis, and rash in
children. Associated findings would be generalized lymphadenopathy and splenomegaly
on physical examination. The lack of these makes the diagnosis unlikely in this case. A
complete blood count and monospot (choice A) can help confirm this diagnosis when it
is suspected.
The rapid antigen detection tests have excellent specificity (>95%) and thus a positive
test does not need to be confirmed by repetition (choice B).
Most practitioners choose to send a swab for bacterial culture (choice C) in the setting
of a negative rapid test in order to increase sensitivity because of the potentially
devastating sequelae of a missed case of streptococcal pharyngitis, but this is
unnecessary with a positive test.
Amoxicillin-clavulanate (choice D) is a combination of a beta-lactam antibiotic and a
beta-lactamase inhibitor. It is useful in the treatment of infections due to organisms that
have developed resistance via the production of a beta-lactamase enzyme. Group A
streptococcus does not produce a beta-lactamase. Thus, this combination is
unnecessary in this case.

The mother of a 3-year-old patient of yours calls the office in the morning to report that
her daughter woke up with a "red eye with a thick yellow discharge." You do not have any

37

open appointments in the morning, and since the girl does not have severe pain or
discomfort and has no change in vision, you schedule an appointment in the afternoon.
You look over the patient's chart and note that she is generally a very healthy child who is
developing normally. At this time you should tell the mother that:
A. Careful hand washing does not affect the spread of this condition
B. She should be excluded from child care until after your examination and
approval for readmission
C. She should take her daughter for a CT scan before the appointment to rule out
orbital cellulitis
D. Since the condition is most likely due to a self-limited virus, the appointment is
for reassurance and not to administer therapy
E. Topical corticosteroids are indicated if the patient has a herpetic dendritic ulcer
Explanation:
The correct answer is B. This patient most likely has conjunctivitis due to a bacterial
infection. Conjunctivitis is a common condition in children that is usually bacterial or
viral. Bacterial infections are usually purulent, while viral infections are not. Topical
antibiotic therapy is typically given for bacterial infections. Conjunctivitis is contagious
and it usually spreads by direct contact. Respiratory spread may occur. Children should
be excluded from childcare settings until they are examined, given treatment if
indicated, and approved for readmission.
It is incorrect to tell the mother that careful hand washing does not affect the spread of
this condition (choice A) because the spread of infection is minimized by hand washing.
It is inappropriate to tell the mother that she should take her daughter for a CT scan
before the appointment to rule out orbital cellulitis (choice C). A CT scan is used to
evaluate a patient with suspected orbital cellulitis, which presents with proptosis,
decreased visual acuity, and pain with movement. It is unlikely that the patient in this
case has orbital cellulitis, and if the symptoms that the mother reported were consistent
with orbital cellulitis, you would want to evaluate her before sending her for diagnostic
studies.
Since the condition is most likely due to a self-limited virus, the appointment is for
reassurance and not to administer therapy (choice D) is incorrect. The "thick yellow
discharge" that the mother described is consistent with a bacterial infection, not a viral
infection. The appointment is for evaluation and the administration of antibiotic therapy if
indicated.
Topical corticosteroids are indicated if the patient has a herpetic dendritic ulcer (choice
E) is incorrect. Corticosteroids are contraindicated in patients with a herpetic dendritic
ulcer because they can worsen the condition. Individuals with a dendritic ulcer often
have a painful, red eye, with visual blurring. Discharge may be present.

A 22-year-old man comes to the office for management of his allergic rhinitis. He has
been told by various physicians over the years that he suffers from this disorder, but that
he has never had any medications prescribed. Rather, he has been instructed to avoid
certain environmental exposures. He reports that each spring and early summer he
routinely suffers from sneezing, nasal discharge, and pruritus as well as eye itching and
watering. The symptoms often abate in the fall and winter, although he occasionally has
symptoms during this time. A survey of his environmental exposures reveals no toxic
irritants in the workplace, perfumes, or colognes that seem to trigger the symptoms.

38

However, cat hair exacerbates his symptoms. He works approximately an hour from his
home and that he commutes by car. The most appropriate next step in management is to
A. advise him to have his home cleaned weekly to decrease the number of
triggers
B. initiate over-the counter antihistamine therapy
C. prescribe an intranasal antihistamine
D. prescribe an intranasal steroid
E. prescribe an oral antihistamine
Explanation:
The correct answer is D. This patient has allergic rhinitis (AR). AR may be seasonal or
perennial with significant overlap. For seasonal rhinitis there are well identified triggers
in spring and summer. Perennial is characterized by more chronic symptoms with
occasional flares. The diagnosis is solely based upon the history and physical
examination. For this patient who appears to have a significant overlap between the two
and has no clear allergen identified, medical treatment is indicated. Medical therapy has
been shown to improve symptoms and is associated with an overall improved quality of
life. Topical intranasal steroids are superior to prescription non-sedating antihistamines
for overall symptom control and for fewer side effects. For this reason, they are now
considered first-line pharmacotherapy for allergic rhinitis.
Since the patient has no clear allergen identifiable, it will likely not benefit him to have
weekly home cleaning (choice A). For patients with such triggers identified, this
maneuver often dramatically improves symptoms.
Over-the-counter antihistamine therapy (choice B) are so-called "sedating" or
"traditional" antihistamines. These are not good agents for this patient because of his
necessity to drive each day. In fact, studies have documented the driving performance
of patients on these drugs are similar to those with alcohol intoxication.
Prescription strength intranasal (choice E) or oral (choice C) antihistamines are so
called "non-drowsy" formulations. These are superior to the standard first generation
agents but are not first-line therapy any longer. Steroids are now considered more
efficacious and should be tried first. The intranasal formulations offer no added benefit
when compared to oral formulations and are therefore not recommended.

A 42-year-old woman who you have been treating for anxiety calls your office demanding
to talk to you . The nurse gets you immediately because she sounds "out of control."
When you finally get to the phone, she is screaming that, "a plumber is working in her
house and he hurt his eye." You ask for the details of the injury and she says that all she
knows is that he "got something in his eye and he is freaking out." You instruct her to look
at the bottle and read the label to you, but all you can understand from her hysterical
speech is, " pH of 12.2." You hear a man in the background screaming in pain. You try to
calmly explain to her that the most appropriate, immediate management is to
A. call an ambulance to bring him to the hospital, and place a patch over the
affected eye
B. call the ophthalmologist and schedule an appointment for the next available
time
C. flush his eye with a substance that she can find in her cabinet with a pH of 6.9
or lower
D. pry the eye open and flush it continuously with cold running water before going
39

to the hospital
E. pry his eye open and vigorously rub the entire eye, especially under the upper
and lower lids
Explanation:
The correct answer is D. This patient most likely has an alkaline eye burn, which is
extremely destructive, and the process of destruction continues as long as the
substance is in contact with the tissues. Immediate removal is essential, and the best
method is massive irrigation. In the emergency room, sterile saline would be used, but
at home, tap water will do. This irrigation is important even before going to the hospital
(because the alkaline fluid will remain in the eye if you leave the irrigation to the
emergency room physicians).
It is extremely dangerous to call an ambulance to bring him to the hospital, and place a
patch over the affected eye (choice A). The alkaline chemical will begin to destroy the
eye if you do not begin immediate irrigation while waiting for help.
Alkaline eye burns are emergencies that require immediate irrigation and medical
attention. Calling the ophthalmologist and scheduling an appointment for the next
available time (choice B) will leave too much time for the chemical to destroy the eye.
It would be inappropriate to tell her to flush his eye with a substance that she can find in
her cabinet with a pH of 6.9 or lower (choice C). Irrigation with cold tap water is the
treatment while waiting for additional medical help (in the ambulance or at the hospital).
While it is necessary to try to remove as much of the chemical as possible, telling her to
pry his eye open and vigorously rub the entire eye, especially under the upper and lower
lids (choice E) is incorrect. Massive irrigation is used to dilute the chemical and wash
away any particles remaining in the eye. Gentle swiping may be appropriate later on in
the treatment to wipe away any little remaining particulate matter, however, this may be
done after irrigation.

40

You might also like